You are on page 1of 75

Mt s phng php

chng minh bt ng thc


Chuyn
Ton hc
Trn Tin t
Lu Trung Kin
Nguyn Hng c
Th Hng Vn
Lp 10 Ton
Trng THPT chuyn Trn Ph
Hi Phng. Thng 5/ 2009

Trang 2



Mt s phng php
chng minh bt ng thc
Phng php SOS
Phng php dn bin
Phng php PQR
Phng php SS







TRN TIN T LU TRUNG KIN
NGUYN HNG C TH HNG VN
----------------------***----------------------
Lp 10 Ton THPT chuyn Trn Ph
Hi Phng. Thng 5/2009


Trang 3

LI NI U
t ng thc l 1 vn kh kh v th v trong ton hc. Trong cc
k thi hc sinh gii ton quc gia, thi Olympic Ton khu vc v quc
t, thi Olympic Ton sinh vin gia cc trng i hc v cao ng, cc bi ton lin
quan n bt ng thc cng hay c cp v thng thuc loi kh hoc rt kh.
Cc bi ton v c lng v tnh gi tr cc tr ca cc tng, tch cng nh cc bi
ton xc nh gii hn ca mt s biu thc cho trc thng c mi quan h t nhiu
n cc tnh ton, c lng bt ng thc tng ng.
Trn th gii mi ngy li c thm rt nhiu bt ng thc thch thc kh nng
t duy v c sng to ca con ngi. L thuyt bt ng thc v c bit, cc bi tp
v bt ng thc rt phong ph v cc k a dng. V vy m khng c g l kh hiu
khi bt ng thc lun c xp vo lp cc bi ton truyn thng ca hu ht cc
cuc thi ton. Khng ch c nhiu ng dng trong i sng, bt ng thc cn cho
chng ta nhng ci nhn mi m, thi thc s t m, say m ni mi con ngi. Chng
ta c bit ti cc bt ng thc c in kh ni ting nh AM - GM, Cauchy
Schwarz, Nesbit, BT hon v Nhng cng vi s pht trin ca ton hc, cc bt
ng thc hin ai cng ngy cng tr nn cht hn i hi s ra i ca cc
phng php mi. Nhm chng ti vit chuyn ny l trnh by v 3 chng minh
c bn cho cc bt ng thc hin i : dn bin, S.O.S, v phng php P.Q.R. y
l 3 phng php kh hiu qu v ph bin chng minh nhiu bt ng thc ngy
nay. Bn cng 3 phng php ny chng ti cng gii thiu c phng php S.S
mt phng php kt hp ca S.O.S v Schur.
Chng ti xin chn thnh cm n thy gio on Thi Sn, ngi giao
cng vic ny v cung cp nhiu ti liu cho chng ti. Trong qu trnh lm chuyn
ny, chng ti c s dng nhng kin thc c trnh by trong cun sch Nhng
vin kim cng trong bt ng thc ton hc ca tc gi Trn Phng, cun Sng
to bt ng thc ca anh Phm Kim Hng, chuyn v phng php dn bin ca
Phan Thnh Vit, chuyn v bt ng thc Schur v phng php i bin pqr ca
V Thnh Vn. Chng ti hi vng chuyn ny s gip bn c phn no trn con
ng n vi bt ng thc.
Tc gi: Lu Trung Kin
Cng nhm cng s
Trn Tin t - Nguyn Hng c - Th Hng Vn
Cng vic ca cc thnh vin
Phn A: Nguyn Hng c
Phn B: Trn Tin t Lu Trung Kin
Phn C: Th Hng Vn
Phn D: Nguyn Hng c Trn Tin t
B

Trang 4

MC LC
LI NI U ............................................................................................ 3
MC LC .................................................................................................. 4
A PHNG PHP SOS ........................................................................ 6
A.I. Gii thiu v phng php SOS .................................................................. 6
A.I.1 . L thuyt ................................................................................................... 6
A.I.2 . Bi tp ...................................................................................................... 7
A.II. Mt s ng dng quan trng ca phng php S.O.S............................ 14
A.III. Phng php S.O.S trong bt ng thc hon v vng quanh. .......... 16
B PHNG PHP DN BIN ............................................................ 18
B.I. Gii thiu v t t ng php dn bin ....................................................... 18
B.II. Bt ng thc i xng 3 bin t cc tr ti tm .................................... 18
B.III. Dn bin bng k thut hm s .............................................................. 25
B.IV. Bt ng thc 3 bin cc tr t ti bin ............................................... 31
B.V. nh l dn bin mnh S.M.V v bt ng thc 4 bin ............................ 34
B.VI. Dn bin bng hm li............................................................................. 38
B.VII. Dn bin khng xc nh - UMV ............................................................. 42
B.VIII. Dn bin ton min (EMV) ....................................................................... 44
B.VIII.1 . EMV vi bin ti 0:.............................................................................. 44
B.VIII.2 . EMV vi bin trong tam gic ............................................................... 45
B.IX. nh l dn bin tng qut {GMV GENERAL MIXING VARIABLES} .. 47
B.X. Bi tp .......................................................................................................... 50
C BT NG THC SCHUR V PHNG PHP BIN I PQR . 52
C.I. Bt ng thc Schur ................................................................................... 52
C.I.1 . L thuyt ................................................................................................. 52
C.I.2 . V d ....................................................................................................... 55

Trang 5

C.II. Phng php bin i pqr ......................................................................... 59
C.II.1 . a thc i xng 3 bin ......................................................................... 59
C.II.2 . Xy dng cc bt ng thc ca pqr ..................................................... 60
C.II.3 . V d ....................................................................................................... 61
D PHNG PHP SS (SCHURS.O.S) ....................................... 67
D.I. Gii thiu...................................................................................................... 67
D.II. Phng php SS vi cc bt ng thc t cc tr ti tm .................... 68
D.III. Phng php SS vi cc bt ng thc cc tr khng t ti tm..... 71
LI KT .................................................................................................. 74
MT S PHNG PHP CHNG MINH BT NG THC
PHNG PHP SOS
Trang 6


A PHNG PHP SOS
A.I. Gii thiu v phng php SOS
A.I.1 . L thuyt
Trc tin chng ta s xem xt v d sau:
CMR:
2 2 2
a b c ab ac bc + + > + +
Gii: BT cn chng minh tng ng vi :

2 1
( ) 0
2
a b >


chng minh bt ng thc trn khng kh, nhng chng ta ch quan tm bn cht thc s
ca cch lm trn. Cch lm trn a bt ng thc thnh dng tng ca cc bnh phng,
li dng tnh cht
2
0 X X R > e chng minh bt ng thc. cng chnh l tng
ca phng php S.O.S( sum of square) a bt ng thc c dng 0 S > thnh dng nh
sau:

2 2 2
( ) ( ) ( ) 0 S b c S c a S a b
a b c
+ + >
Nu cha lm quen vi S.O.S bn s ch gii quyt c bi ton khi m
, , 0 S S S
a b c
> .Nhng S.O.S c th gii quyt trng hp m 1 trong cc s
, , S S S
a b c
m (
v hu ht cc trng hp u xy ra nh vy).
C th chng ta c:
Gi s S =
2 2 2
( ) ( ) ( ) 0 S S b c S c a S a b
a b c
= + + > (*)
Nu
, , 0 S S S
a b c
>
th S 0 >
Nu a b c > > v
, , 0 S S S S S
b b a c b
+ + >
th S 0 >
Nu a b c > > v
, , 2 , 2 0 S S S S S S
a c b a c b
+ + >
th S 0 >
Nu a b c > > v
, 0 S S
b c
>
,
2 2
0 a S b S
b a
+ >
th S 0 >
Nu 0, 0 S S S S S S S S S
a b c a b b c c a
+ + > + + > th S 0 >
Nhn chung trong phng php S.O.S vic chng minh cc iu kin trn l kh n gin nu
a c BT v dng (*).
Phn kh nht trong phng php S.O.S l lm th no a bt ng thc v dng (*).
y chnh l th to ra nt c o ca phng php S.O.S khi m bn s thy rng cch biu
din di dng (*) khng phi l duy nht. Sau y l mt s cng thc c bn hay dng
cc bn thun tin hn khi chng minh BT bng phng php ny:
Phn
A
MT S PHNG PHP CHNG MINH BT NG THC
PHNG PHP SOS
Trang 7



A.I.2 . Bi tp
c th ha cho iu ny chng ta hy xem xt cc v d sau
Bi A.I.1: Cho a,b,c l cc s dng . CMR:
2 2 2
( )( )( ) 4( )
12
a b b c c a ab bc ca
abc a b c
+ + + + +
+ >
+ +

Chng minh
( )
( )
( ) ( )( )
( )
( ) ( )
2
2 2
2
2
3 3
2
2 2 2
2 2
2 2 2
2
2
3
1
2
1 1
3 3
cyc
cyc
a b ab a b
a b a b
b a ab
a b ab a b a b a b
a b c ab bc ca a b
a b c a b c a b
+ =

+ =
+ + = +
+ + =
+ + + + =



( )( )( ) ( )
( )
( ) ( )
( )
( )
( )
( )( )
( )
( ) ( )
( )( )
2
2
3 3 3
2
2 2
2 2
2
2
2 2
2
3 2
2
4 3 2 2
3
8
1
3
2
2
2
3
2 2
1
3
1
2
3
1
3 2
4
cyc
cyc cyc
cyc sym
sym sym cyc
cyc sym sym
cyc sym sym
sy
a b b c c a abc a b c
a b c abc a b c
a b
a b a b
a b a b
a b a
b c a c b c
a b ab a b
a a b a b a b
a a b a ab b a b
a b
+ + + =
+ + =

+ + =
+ + +

=
+ + +
=
= +
= + +






( )
( ) ( )
3
3
2 2
4 2 2
1
3
1
2
m sym cyc sym
cyc cyc cyc
ab a b a
a a b a b a b
=
= +



MT S PHNG PHP CHNG MINH BT NG THC
PHNG PHP SOS
Trang 8

2 2 2
2 2 2
2 2 2
2
2
2 2 2
2 2 2
( )( )( ) 4( )
12
( )( )( ) 4( ) 4( )
8 0
( ) 2 ( )
0
( ) ( ) ( ) 0
( )
cyc cyc
a b b c c a ab bc ca
abc a b c
a b b c c a ab bc ca a b c
abc a b c
a b c b c
abc a b c
S b c S c a S a b
a b c
+ + + + +
+ >
+ +
+ + + + + + +
+ >
+ +

>
+ +
+ + >


Trong :
2 2
2 2 2 2 2 2
2 2
2 2 2 2 2 2
2 2
2 2 2 2 2 2
1 2 ( )
( )
1 2 ( )
( )
1 2 ( )
( )
a b c
S
a
bc a b c bc a b c
b c a
S
b
ca a b c ac a b c
c a b
S
c
ab a b c ab a b c
+
= =
+ + + +
+
= =
+ + + +
+
= =
+ + + +

D thy: S
a
, S
b
, S
c
u l cc s dng. Vy bt ng thc c chng minh. Du bng
xy ra khi a b c = =
Bi A.I.2: Cho a,b,c l cc s dng. CMR
2 3 3 3
2 2 2
3( )
sym
a a b c
b a b c
+ +
>
+ +


Chng minh
2 3 3 3
2 2 2
2 3 3 3
2 2 2
2 2
2 2
2 2 2
2
2
2 2 2
2 2 2
3( )
3( )
( )
( ) ( )
2
( )
( ) ( )
( )
( ) ( ) ( ) 0
( )
sym
sym cyc cyc
cyc
sym
sym sym
a a b c
b a b c
a a b c
a a
b a b c
a a b b c
a ab b
b a b c
a b a b
a b
b a b c
S b c S c a S a b
a b c
+ +
>
+ +
+ +
>
+ +
+
+
>
+ +
+
>
+ +
+ + + >



trong :
2 2 2
2 2
2 2
a
b
c
b a c
S
c
b c ab
S
a
a c ab
S
b
+
=
+
=
+
=

Trng hp 1: a b c > >
D thy S
a
, S
c
> 0. Ta cn CM: 2 , 2 0
a c a b
S S S S + + >
MT S PHNG PHP CHNG MINH BT NG THC
PHNG PHP SOS
Trang 9

Xt
3 3 3
3
2 2 2
3
2
2 3
2 2 3
2
2 3
54
3 2 0
( )
( ) ( ) 6 ( )
0
2 ( )
1 1 1 6
( ) ( 0
2 4 ( ) 4 ( )
2 ( ) ( ) ( ) 24
( ) ( ) 0
2 4 ( ) 4 ( )
a b c abc
abc a b c
a b c a b c a b c
abc a
b c a
b c
abc bc a b c bc a b c
b c a a b c b c a b c abc
b c
abc bc a b c bc a b c
+ +
+ >
+ +
+
>
+
+ + >
+ + + +
+ + + + + +
+ + >
+ + + +


m
3 2 2 3
2 2 ( ) a b c b a c abc a b c + > > > >
2 0
a b
S S + >
Xt
2 2 3 2 2 3
( ) ( 2 ) 2
2
a c
a b c b b a c abc a c c
S S
bc
+ + + +
+ =
m
3 2 2 3
2 2 ( ) b a c a b c abc a b c + > > > >
2 0
a c
S S + >
Trng hp 2: c b a > >
D thy , 0
a c
S S > . Ta cn CM: 2 , 2 0
a c a b
S S S S + + >
Bng cch lm nh trn ta cng s chng minh c 2 , 2 0
a c a b
S S S S + + >
Vy bt ng thc c chng minh. Du bng xy ra khi a b c = =
Bi A.I.3: Cho a,b,c l cc s dng. CMR:
3 3 3
3
54
5
( )
a b c abc
abc a b c
+ +
+ >
+ +

Chng minh
Bt ng thc tng ng vi
3 3 3
3
2 2 2
3
2
2 3
2 2 3
2
2 3
54
3 2 0
( )
( ) ( ) 6 ( )
0
2 ( )
1 1 1 6
( ) ( 0
2 4 ( ) 4 ( )
2 ( ) ( ) ( ) 24
( ) ( ) 0
2 4 ( ) 4 ( )
a b c abc
abc a b c
a b c a b c a b c
abc a
b c a
b c
abc bc a b c bc a b c
b c a a b c b c a b c abc
b c
abc bc a b c bc a b c
+ +
+ >
+ +
+
>
+
+ + >
+ + + +
+ + + + + +
+ + >
+ + + +


(lun ng).
Vy bt ng thc c chng minh. Du bng xy ra khi a b c = =
MT S PHNG PHP CHNG MINH BT NG THC
PHNG PHP SOS
Trang 10

Bi A.I.4: Cho , , 0 a b c > .CMR
( )
( )
2
2
2 3 cyc
a b c a
b c
ab bc ca
+ +
>
+
+ +


Chng minh
Ta c:
( )
( )
( )
( )
( )
( )
( ) ( )
( )
( )
( )
2
2
2
2
2
2
2 3
2 2
2 3
1
4 2
3
3
2 2
4 4 2
3
cyc
cyc
cyc
sym
a b c a
b c
ab bc ca
a b c a a b c a b c
b c
ab bc ca
a b c a b c a b c
b c
ab bc ca
a b c ab bc ca
a b c b a c a b c
a b
b c c a
ab bc ca
a b
+ +
>
+
+ +
+ + + + + +
>
+
+ +
| |
| | + + + + +
|
>
|
| +
+ + \ .
\ .
| |
+ + + + | |
+ + + + + +
| >
|
|
| + +
+ +
\ .
\ .

( )( )
( ) ( )
( ) ( )
( )
2
3 2 2 2
4 2
3 3 sym
a b c ab bc ca a b c a b c
b c c a
ab bc ca a b c ab bc ca
| |
+ + + + + + + +
|
>
|
+ +
+ + + + + + + |
\ .


( )
( )( )
( )
( ) ( )
( )
( )
( )( )
( ) ( )
( )
2 2
2
1
2
3 3
1 1
2 3 3
sym sym
sym
a b a b
b c c a
ab bc ca a b c ab bc ca
a b
b c c a
ab bc ca a b c ab bc ca

>
+ +
+ + + + + + +
| |
|

|
+ +
+ + + + + + + |
\ .


( )
2
0
a
sym
S b c >


trong :
( )( )
( ) ( )
( )
( )( )
( ) ( )
( )
( )( )
( ) ( )
( )
1 1
2 3 3
1 1
2 3 3
1 1
2 3 3
a
b
c
S
a b c a
ab bc ca a b c ab bc ca
S
a b b c
ab bc ca a b c ab bc ca
S
b c c a
ab bc ca a b c ab bc ca
=
+ +
+ + + + + + +
=
+ +
+ + + + + + +
=
+ +
+ + + + + + +

D thy , 0
b c
S S >
Ta s chng minh 0
a b
S S + > (theo tiu chun 2).
Tht vy:
( )( ) ( )( )
( ) ( )
( )
1 1 1
3 3
a b
S S
a b c a a b b c
ab bc ca a b c ab bc ca
+ = +
+ + + +
+ + + + + + +


MT S PHNG PHP CHNG MINH BT NG THC
PHNG PHP SOS
Trang 11

( ) ( )
( )
( )
( ) ( ) ( )
( ) ( ) ( ) ( )
( )
2
2
3 3
1
0
3 3
a b
ab bc ca ab bc ca ab bc ca a b c b
a b c a
a b b c ab bc ca
S
a b c ab b c
S
c a
| |
+ + + + + + + + +
|
\ .
= + >
+ +
+ + + + + + + + +
+

Vy bt ng thc c chng minh. Du bng xy ra khi a b c = =
Bi A.I.5: Cho a,b,c l cc s dng . CMR:
( )
2 2 2
2 2 2
3
2
a b c
a b c
a b b c c a
+ +
+ + >
+ + +

Chng minh
Bt ng thc cn chng minh tng ng vi:
( )
2 2 2
2 2 2
3
2 2 2
a b c
a b c a b c a b c
a b b c c a
+ +
+ + + +
+ + >
+ + +
(1)
Xt v tri (1)
( ) ( )
( )( )( )
( )( ) ( )
( )( )( )
( )( ) ( ) ( )( ) ( )
( ) ( )( ) ( )
( )( )( )
( )( ) ( )
2 2 2
2
2 2 2 3
2 2 2 3
2
2
8 8
2 2
4
sym cyc
cyc cyc cyc cyc cyc
cyc cyc cyc
a b c a b c
a b b c c a
a b c c a a b b c c a a
a b b c c a a b b c c a
a bc a b a b abc a a b b c c a abc a
a b b c c a a b b c c a a b b c c a
a bc a b a b
abc a b
a b b c c a
+ +
+ +
+ + +
+ + + + +
=
+ + + + + +
+ + + + +
=
+ + + + + + + + +
+ +
+
=
+ +
=
+



( )
( )( ) ( )
( ) ( )
( ) ( )( )
( )
( ) ( )( )
( ) ( )
( )( )( )
( )
( )( ) ( )
( )
( )( ) ( )
( ) ( )
( ) ( )( )
( )
( )( ) ( )
2
2
2 2 3
2 2
2 2
2 2
2
3
2
3 3
2
2 2
cyc
cyc cyc cyc
cyc cyc cyc
cyc
a b c a b c
c
a b b c c a a b b c c a
a b a b abc a b c a b c a b c
a b b c c a a b b c c a
a b abc a b c ab a b a b c a b c
a b b c c a a b b c c a a b b c c a
a b abc a b c
a b b c c a
+ +
+

+ + + + + +
+ + + + +
=
+ + + + + +
+ + + +
= +
+ + + + + + + + +
+ +
=
+ + +

( )
( ) ( )( )
( ) ( )
( )( ) ( )
2 2
3
2 2
cyc cyc
bc b c a b c a b c
a b b c c a a b b c c a
+ +
+
+ + + + + +


( )
( )( )( )
( )
( ) ( ) ( )
( ) ( )
( ) ( ) ( )
( )
( )
( )( )( )
2 2 2
2
2
2
3 2
2 2 2
3 2
2
cyc cyc cyc
sym
a b c bc b c a b c a b c
a b b c c a a b b c c a a b b c c a
a bc a b c a
b c
a b b c c a
+ +
= +
+ + + + + + + + +
+ + +
=
+ + +


Xt v phi (1)
MT S PHNG PHP CHNG MINH BT NG THC
PHNG PHP SOS
Trang 12

( )
( ) ( )
( ) ( )
( ) ( )
( )
( )
( ) ( )
( )
2 2 2
2 2 2
2
2 2 2
2 2 2
2
2 2 2
3
2 2
3
2
3
2 3
2 3
cyc
a b c
a b c
a b c a b c
a b c a b c
a b c a b c
b c
a b c a b c
+ +
+ +

+ + + +
=
+ + + +
=
+ + + + +

=
+ + + + +
=


Vy ( ) 1
( )
( )
( )( )( )
( )
( ) ( )
( )
( )
( )
( ) ( ) ( )
( ) ( )
( )
( )
2
2
2
2 2 2
2
2
2 2 2
2
3 2
2
2 3
3 2 1
0
2
2 3
0
sym cyc
sym
a
cyc
a bc a b c a b c
b c
a b b c c a
a b c a b c
a bc a b c a
b c
a b b c c a
a b c a b c
S b c
+ + +
>
+ + +
+ + + + +
| |
+ + +
|
>
|
+ + +
+ + + + +
|
\

.
>


Trong :
( )( )( )
( ) ( )
( )
( )( )( )
( ) ( )
( )
( )( )( )
( ) ( )
( )
2
2 2 2
2
2 2 2
2
2 2 2
2 2 1
3
2 2 1
3
2 2 1
3
a
b
c
a bc ab ac
S
a b b c c a
a b c a b c
b ca ab bc
S
a b b c c a
a b c a b c
c ab bc ac
S
a b b c c a
a b c a b c
+
=
+ + +
+ + + + +
+
=
+ + +
+ + + + +
+
=
+ + +
+ + + + +

Ta d dng chng minh c , 0
a c
S S >
Theo tiu chun 3 ta cn chng minh 2 0 & 2 0
a b c b
S S S S + > + > (Phn chng minh ny
n gin, xin dnh cc bn t chng minh)
Bi A.I.6: Cho a,b,c l cc s dng .CMR:
( )
( )
2 2 2
2 2 2
6 a b c
a b c
a b c
b c a a b c
+ +
+ + + + + >
+ +

Chng minh
Bt ng thc cn chng minh tng ng vi:
MT S PHNG PHP CHNG MINH BT NG THC
PHNG PHP SOS
Trang 13

( )
( )
( )
( )
( ) ( )
( )
( )
( )
( )
( )
2 2 2
2
2
2 2 2
2 2
2
2
2
2
6
2
2 3
2
1 2
0
0
0
0
sym
sym
sym cyc
sym
sym
sym
a
cyc
a b c
a
a a b c
b a b c
a b a b c
a b c
b a b c
a b a b
b a b c
a b
b a b c
a c b
a b
b a b c
a c b
a b
b
S b c
+ +
| |
> + +
|
+ +
\ .
| | + +
> + +
|
+ +
\ .

>
+ +
| |
>
|
+ +
\ .
| |
+
>
|
|
+ +
\ .
+ | |
>
|
\ .

>


trong
a
b
c
a b c
S
c
b c a
S
a
a c b
S
b
+
=
+
=
+
=

( ) ( ) ( )
( ) ( ) ( )
2 2 2 2 2 2
3 6 3 3
0
0
a b
a b c
cyc
a b b a
b c c a
c c
a b
S S S
b a
a b c ab a c b ac b a a bc
S S S S S S
abc
ba bc abc ab ac abc ca cb abc
S S S S S S
abc
+ + + + +
+ + =
| |
+ + = + > = >
|
\ .
+ + + + +
+ + =
>

theo
tiu chun 5 bt ng thc c chng minh. Du bng xy ra khi a b c = =
C th ni trong 5 iu kin ca S.O.S th iu kin 5 l kh p dng nht, c bit l khi
nhng h s , ,
a b c
S S S phc tp. Tuy nhin trong bi ton va ri p dng iu kin 5 l
hp l nht (cc iu kin cn li p dng s rt phc tp v c th khng lm c). V
vy mt iu ht sc cn thit l chng ta phi p dng 5 iu kin c bn mt cch tht
linh hot, hp l. C nh vy bn mi c th s dng c ton b sc mnh ca S.O.S
Bi A.I.7: (IMO 2005 ) Cho a,b,c > 0 tha mn 1 abc > CMR:
5 2 5 2 5 2
5 2 2 5 2 2 5 2 2
0
a a b b c c
a b c b c a c a b

+ + >
+ + + + + +
(*)
Chng minh
MT S PHNG PHP CHNG MINH BT NG THC
PHNG PHP SOS
Trang 14

Cc phng php CM bt ng thc hin i ( c bit l Schur, PQR, S.O.S) u rt xem
trng vic ng bc ha bt ng thc( qua cc v d trn hn cc bn u hiu c tm
quan trng ca cng vic ny).
Vic ng bc ha (*) c thc hin thng qua nh gi sau y
( ) ( )
( )
( )
4 2 2 2
5 2 5 2 4 2
2 5 2 2 5 2 2 4 2 2
4 2 2
2
2
a a b c
a a a a abc a a bc
a b c a b c abc a bc b c
a b c
+

> = >
+ + + + + +
+ +

nh gi trn c th xem nh cha kha ca bi ton ny bi v phn CM cn li rt n gin
:
t
2 2 2
, , x a y b z c = = =
( )
( )
( )
( )
( )
( )
( )
( )
( )
2 2 2 2
2
2 2 2 2 2 2
2 2 2
2
, ,
2 2 2
( ) ( ) ( ) 0
cyc cyc
x x y z z z x y x xy y
f x y z x y
x y z x y z y x z
S y z S z x S x y
x y z
+ + + + +
= =
+ + + + + +
= + + >


A.II. Mt s ng dng quan trng ca phng php
S.O.S
Mt ng dng quan trng ca S.O.S l sng to ra cc bt ng thc mi. thng
l nhng bt ng thc rt mnh, kh chng minh bng cc bt ng thc thng thng(
Cauchy, BCS...).Sau y l mt s v d quan trng v in hnh nht.
V d A.II.1: Tm hng s k tt nht BT sau ng vi mi a,b,c dng:
2 2 2
3
2
a b c ab bc ca
k k
b c c a a b a b c
+ +
+ + + > +
+ + + + +

S khc bit ln nht gia ng thc v bt ng thc l du , > s . iu ny th hin
r khi bn chng minh bi ton trn bng phng php thng thng: x l
2 2 2
ab bc ca
a b c
+ +
+ +
v
du >iu bt ng l S.O.S li gii quyt iu ny rt d dng.
Chng minh
Bt ng thc cn chng minh tng ng vi:
( )
( )( )
( )
( )
( )( )
2 2 2
2 2
2 2 2
2 2 2
2
2 2 2
1
1
2
1 1
2 2
0
( ) ( ) ( ) 0
cyc
cyc cyc
cyc
a ab bc ca
k
b c a b c
a b a b
k
b c a c a b c
a b c
a b k
b c a c
S b c S c a S a b
a b c
+ + | | | |
>
| |
+ + +
\ . \ .

>
+ + + +
(
+ +
>
(
+ +
(

+ +

>


MT S PHNG PHP CHNG MINH BT NG THC
PHNG PHP SOS
Trang 15

trong
( )( )
( )( )
( ) ( )
2 2 2
2 2 2
2 2 2
a
b
a
a b c
S k
a b a c
a b c
S k
a b b c
a b c
S k
c b a c
+ +
=
+ +
+ +
=
+ +
+ +
=
+ +

iu kin cn: Cho c b a = = ta c
( )
( )
( )
( )
( )
( )
2 2
2
2
2 2 2
2
2
2
0
2
2
2 2
, ( )
2 2 2
2 1
2 2 2
2 2 2 0
a b
a b k
b a b
a
a b t a b
k u a b t
a b a b t b
b
t u t
f t t ut u
(
+
>
(
+
(

| |
+
|
+ +
\ .
s = = = =
+ + | |
+
|
\ .
+ = +
= + =

( ) 0 f t = c nghim
2
3 1
' 2 2 0
2
u u u

A = + > >
iu kin :
Ta chng minh
3 1
2
k

= l hng s tt nht ca bt ng thc trn.
( )
( )( )
2 2 2
2
0
cyc
a b c
a b k
b c a c
(
+ +
>
(
+ +
(

(1)
Khng mt tnh tng qut gi s a b c > >
a b c
S S S s s
( )( ) ( )( )
2 2 2 2 2 2
2
a b
a b c a b c
S S k
a b a c a b b c
+ + + +
+ = +
+ + + +

t
2
a b
v
+
=
( )
( )
( )( )
( )( )
( ) ( )
( )
2
2
2 2
2 2
2
2 2
2
2
2
2 2 2
2
2
a b
a b
c v c
v c v c
v c
S S k k k
v b c a c v v c
b c c a
v
(
+
+ + (
+ +
(
+

+ > > =
+ + +
( + + +
(


( )
( )
2 2
2
2 2 , 0
2
a b
v c
S S k u v c k
v v c
| |
+
( + > = >
|

|
+
\ .
(do ( )
3 1
,
2
u v c k

> = )
Do
a b c
S S S s s v 0
a b
S S + > nn 0, 0, 0
b a b c b
S S S S S > + > + > .
MT S PHNG PHP CHNG MINH BT NG THC
PHNG PHP SOS
Trang 16

Theo tiu chun 2 suy ra
3 1
2
k

= th (1) ng vi mi , , 0 a b c >
Kt lun:
3 1
2
k

= l hng s tt nht bt ng thc
2 2 2
3
2
a b c ab bc ca
k k
b c c a a b a b c
+ +
+ + + > +
+ + + + +
ng vi mi , , 0 a b c >
Mt cu hi mun thu: th no l mt cch gii hay cho mt bi ton? Mt bi ton c
th c nhiu cch gii, mi cch gii mang theo mt mu sc ring, mt nt c o
ring. Chnh v th kh c th c mt cu tr li chnh xc cho cu hi trn v mi ngi
quan nim ci hay mt cch khc nhau (mt li gii hay l mt li gii ngn nhng cng
c th l mt li gii l mt s sng to c o, mang theo mt tng mi, t tng
mi). Theo nhng tiu chi trn th li gii cho bi ton trn cha phi l mt li hay.
Mc d vy nhng li gii trn mang tnh hiu qu.
Tm hng s k tt nht bt ng thc sau ng:
( )
2 2 2
1 1 1
9
ab bc ca
a b c k k
a b c a b c
+ + | |
+ + + + + > +
|
+ +
\ .

R rng theo phng php S.O.S, hai bi ton ny ging nhau v mt bn cht. ngh
ra mt cch lm c o cho 2 bi trn l chuyn khng d ch ng ni l tm mt li
gii hay p dng cho tt c cc bi ton c dng trn. Nhng khi m bt ng thc pht
trin nhanh vi tc chng mt nh hin nay, th nhng phng php c in hn cha
lm hi lng nhng ngi yu thch bt ng thc. C th S.O.S khng mang li mt li
gii p mt nh BT Cauchy hay cc phng php chng minh BT c in khc,
nhng n gii c mt lp ln cc bi ton hay v kh, cng chnh l ng dng ln
lao m S.O.S mang n cho chng ta
A.III. Phng php S.O.S trong bt ng thc
hon v vng quanh.
Chng minh bt ng thc hon v vng quanh lun kh hn bt ng thc i xng.
Nhn chung khi chng minh BT hon v chng ra thng gi s ( ) ax , , a m a b c = hay ni
cch khc l xt 2 trng hp & a b c a c b > > > > . Cn trong phng php SOS, chng ta s
xt 2 trng hp & a b c c b a > > > > . Ch cn nh k 5 iu kin ca S.O.S, chng ta u d
dng l gii iiu ny.
V d A.III.1: Cho a,b,c l 3 cnh tam gic CMR 3 2 3
a b c a c b
b c a c b a
| | | |
+ + > + + +
| |
\ . \ .

Phng php gii ca bi ny( v nhiu bi BT hon v khc ) l quy ng mu s a v
dng ng bc. Rt n gin nhng hiu qu th tht khng ng!
Chng minh
Bt ng thc cn cng minh tng ng vi:
MT S PHNG PHP CHNG MINH BT NG THC
PHNG PHP SOS
Trang 17

( ) ( )
( ) ( ) ( ) ( ) ( )
( ) ( ) ( )( ) ( )( )
2 2 2 2 2 2
3 3 3 2 2 2 3 3 3 2 2 2 3 3 3
2 2 2
2 2 2
3 2 3
3 2 3
2 1
2 2
3 2
( ) ( ) ( ) 0
sym sym cyc
a c b a c b b c c a a b abc
a b c a c b a c b a b c b c c a a b a b c abc
b a a b a b a b a b c a b
S b c S c a S a b
a b c
+ + > + + +
( (
+ + + + s + + + + + + +

+ s + + + +
+ +

>

trong
5 5 3
5 5 3
5 5 3
a
b
c
S b c a
S c a b
S a b c
= +

= +

= +


Mc d cc h s , ,
a b c
S S S kh n gin nhng chng ta khng th ch dng 5 iu kin c
bn lm c bi ton ny.
B : Nu
( )
2
2
1 0
a c b
S S S + + + >
0 0
1 1
0 0
c b a b
c b a b
a b c b c b a b
S S S S
S S S S




> >

+ > v + >


+ +
+ > + >


th 0 S >
p dng: Khng mt tnh tng qut gi s b nm gia a v c. Xt 2 trng hp:
1. c b a > > . Khi 5 5 3 0; 8 12 0; 8 2 0
b a b b c
S c a b S S b a S S c b = + > + = > + = >
Theo iu kin 2 suy ra pcm
2. a b c > > . Khi 4 18 12 12 0
a c b
S S S c b a + + = + >
- Nu a b c b > th 0 2 c a b b c c b > > > >
( )
8 2 4 2 0
2 2 5 0
b c
c b b
S S c b b b
S S S b c a b
+ = > >
+ > + = + >

- Nu c b a b > th
( )
0 2
8 2 4 2 0
2 2 5 3 3 3 0
a b
a b b
b c a b b a c a
S S b a a a
S S S a c a b c a b
> > > + >
+ = > >
+ > + = + > + >

S dng b vi 1 = suy ra pcm.

MT S PHNG PHP CHNG MINH BT NG THC
PHNG PHP DN BIN
Trang 18


B PHNG PHP DN BIN
B.I. Gii thiu v t t ng php dn bin
Phng php dn bin l phng php dng chng minh nhng bt ng thc trong
cc trng hp ng thc xy ra khi tt c cc bin s bng nhau (cc tr t ti tm), c mt
s bin bng nhau (cc tr t c c tnh i xng) hay khi c mt bin nm trn bin (cc
tr t c ti bin). tng chung l: nu ta a c v trng hp c 2 bin bng nhau,
hoc mt bin c gi tr ti bin, th s bin s gim i. Do , BT mi n gin hn BT
ban u, c bit nu BT mi ch cn 1 bin th bng cch kho st hm s 1 bin ta s
chng minh c BT kh n gin. Chnh v t tng l gim dn s bin nn phng
php ny c gi l phng php dn bin.
Hu ht cc BT u ri vo 1 trong cc dng ni trn, nn c th thy ng dng ca
phng php dn bin l rt ln, c bit l khi chng minh bt ng thc 3 hay 4 bin. Vi
s bin t nh vy, sau khi dn bin vic chng minh s n gin hn. Tuy nhin, phng
php dn bin cng c th c p dng trong cc trng hp n bin tng qut, bng cch s
dng nhng nh l v dn bin tng qut mt cch linh hot.

B.II. Bt ng thc i xng 3 bin t cc tr ti
tm
ng dng t tng ca phng php dn bin vo chng minh BT 3 bin i xng cc tr
t ti tm, ta rt ra cch lm sau
Gi s cn chng minh f ( x, y, z) 0 vi x, y, z tha mn tnh cht no y.
Bc 1: (K thut dn v 2 bin bng nhau) nh gi f (x, y, z) f (t, t, z) vi t l bin sao
cho (t, t, z) tha mn mi tnh cht ca (x, y, z)
Bc 2: nh gi f (t, t, z) 0.
Ch : Trong cc bt ng thc ng bc, ta thng dng k thut chun ha li
gii ngn gn v n gin hn.
Chng ta s p dng t tng trn vo mt s v d sau.
V d B.II.1: Cho x,y,z 0. Chng minh rng
3
3 x y z xyz + + > (1)
Chng minh
t
3
( , , ) 3 , , 0 f x y z x y z xyz x y z = + + >
Cch 1:
Phn
B
MT S PHNG PHP CHNG MINH BT NG THC
PHNG PHP DN BIN
Trang 19

Bc 1: Chng minh ( , , ) ( , , ) f x y z f t t z > vi
2
x y
t
+
= . Ta c: t
2
xy suy ra:
( )
3 3 2 2
3 3
( , , ) ( , , ) 3 2 3 3 0 f x y z f t t z x y z xyz t z t z t z xyz
(
= + + + = >


Bc 2: Chng minh ( , , ) 0 f t t z >
Tht vy, ( ) ( ) ( )
3 2
2
( , , ) 0 2 27 0 8 0 f t t z t z t z t z t z > + > + > ( lun ng vi t, z 0)
Vy ta c iu phi chng minh. Du = xy ra khi a = b = c
Cch 2:
Bc 1: Chng minh ( , , ) ( , , ) f x y z f t t z > vi t xy = . Ta c 2t x + y suy ra:
3 3
( , , ) ( , , ) 3 2 3 2 0 f x y z f t t z x y z xyz t z xyz x y t
(
> = + + + = + >


Bc 2: Chng minh ( , , ) 0 f t t z >
Tht vy, ( ) ( ) ( )
3 2
2
( , , ) 0 2 27 0 8 0 f t t z t z t z t z t z > + > + > ( lun ng vi t, z 0)
(pcm) Du = xy ra khi a = b = c
y ta nhn thy bt ng thc l ng bc nn li gii bi ton c th ngn gn hn
bng vic dng k thut chun ha
Cch 3:
Bc 1: Chun ha x + y + z = 1
3
(1) 1 3 1 27 0 xyz xyz > >
Bc 2: t ( , , ) 1 27 f x y z xyz = v
2
x y
t
+
= . Ta c
2
t xy > suy ra
2
( , , ) ( , , ) 27 ( ) 0 f x y z f t t z z t xy = >
Vy ( , , ) ( , , ) f x y z f t t z >
Bc 3: Chng minh ( , , ) 0 f t t z >
Thay z = 1 2t ta c
( ) ( )( )
2
2
( , , ) 1 27 1 2 1 6 1 3 f t t z t t t t = = + ng do t 0
Vy ta c iu phi chng minh. Du = xy ra khi a = b = c
Cch 4:
Bc 1: Chun ha xyz = 1
(1) 3 3 0 x y z x y z + + > + + >
Bc 2: t ( , , ) 3 f x y z x y z = + + v t xy = . Ta c 2t x + y suy ra
( , , ) ( , , ) 2 0 f x y z f t t z x y t = + >
Vy ( , , ) ( , , ) f x y z f t t z >
Bc 3: Chng minh ( , , ) 0 f t t z >
Thay
2
1
z
t
= ta c
( ) ( )
2
2 2
1 2 1
1
( , , ) 2 3 0
t t
f t t z t
t t
+
= + = > ng do t 0
MT S PHNG PHP CHNG MINH BT NG THC
PHNG PHP DN BIN
Trang 20

Vy ta c iu phi chng minh. Du = xy ra khi a = b = c
Mt vi nhn xt:
Trn y l mt v d m u cho php dn bin, vi bin t bng trung bnh cng hay
trung bnh nhn. y cng l nhng k thut chn bin t hay dng Bi ton ch nhm
mc ch cho chng ta ci nhn u tin v ng dng ca t tng php dn bin vo
chng minh bt ng thc.
Cch gii 3 v 4 s dng k thut chun ha, gip li gii c v ngn gn hn.Vic
chun ha cc bt ng thc ng bc cng l mt k thut hay gp trong chng minh
bt ng thc. Nu mt bi ton chun ha(tc l BT c iu kin) th n s "gi "
cho chng ta cch dn bin (phi m bo iu kin), tuy nhin, ngc li mt bi ton
cha chun ha (BTkhng iu kin) th chng ta s c nhiu cch dn bin hn (ni
chung, ta s chn cch dn bin sao cho bo ton c "nhiu" biu thc nht trongBT
- iu ny cng tng ng vi chun ha sao cho biu thc c dngn gin nht). Do
, mt s phi hp tt gia k thut chun ha v dn bin l mt iu cn thit. Tuy
nhin, khi quen vi nhng iu ny th cc bn s thy khng c s khc bit ng k
no gia chng.
S dng phng php dn bin trong chng mnh bt ng thc Cauchy trn rt n
gin do khng kh chng minh f (x , y , z) f ( t , t , z ) 0. Tuy nhin, thc ra cc bi
ton thng khng bao gi n gin nh vy. Chng ta s xt tip nhng v d khc
thy hiu qu ca phng php dn bin.
V d B.II.2: [MOSP 2001] Cho , , 0 a b c > v 1 abc = .CMR:
( )( )( ) ( ) 4 1 a b b c c a a b c + + + > + +
Chng minh
t
( )( )( ) ( ) ( , , ) 4 1 f a b c a b b c c a a b c = + + + + +
( ) ( ) ( ) ( )
2 2 2
2 4 1 abc a b c a b c bc b c a b c = + + + + + + + +
Khng mt tnh tng qut ta gi s { } max , , a a b c =
C
( ) ( )
2 2
2
( , , ) ( , , ) 4 d f a b c f a bc bc b c a a b c bc
(
= = + + +
(


Xt:
( ) ( )
2 2
2 2 2
2 2
1 1 1 1 1 2
4 4 4
1 4 1
4 0
A a a b c bc a b c a
bc a a b c bc
a a
a a bc
| |
= + + + = + + + = + + + +
|
\ .
> + + > + >
(Do { } ax , , a m a b c = v 1 abc = .nn
4
1 4 0 bc
bc
s > )
Vy suy ra 0 d > hay ( , , ) ( , , ) f a b c f a t t > (vi 0 a t > > v
2
1 at = )
C
3 2 2 3
3 2
2 4
( , , ) 2 2 4 4 8 4 2 8 8 f a t t t a t at a t t t
t t
= + + + = + + (do
2
1
a
t
= )

MT S PHNG PHP CHNG MINH BT NG THC
PHNG PHP DN BIN
Trang 21

2
3 2
1 1 1
2(1 ) 4 1 0 t t t
t t t
| |
= + >
|
\ .
(do 1 t s )
Vy ( , , ) ( , , ) 0 f a b c f a t t > > pcm
Du = xy ra khi a=b=c=1
y, chng ta khng th chng minh trc tip ( , , ) ( , , ) f a b c f a t t > m phi nh gi
thng qua vic coi a = max{a,b,c} Vic sp xp th t ca cc bin cng l mt k thut
thng gp trong chng minh bt ng thc. Nhc li l nu BT 3 bin i xng th ta
c th gi s a b c (hoac a b c), cn trong trng hp BT 3 bin hon v vng
quanh th ta c th gi s a = min{a, b, c} (hoc a = max{a, b, c}).
Phng php dn bin thng cng c dng nhiu trong cc bt ng thc c iu
kin. Vi cc BT c iu kin, ta s ko dn bin nh thng m phi lm sao iu
kin lun c tho mn. Chng hn vi K 3 xy yz zt + + = , khi chng ta mun dn 2
bin chng hn y v z bng nhau, tc l ( , , ) ( , , ) f x y z f x t t > th bin t y khng phi
l 1 i lng trung bnh ca y, z m l i lng tho mn
2
2 1 xt t + = . Ta xt mt vi v
d sau.
V d B.II.3: (VMO) Cho x, y, z l cc s thc tho mn
2 2 2
9 x y z + + = Chng minh rng:
2( ) 10 x y z xyz + + s
Chng minh
t f(x,y,z) = 2( ) x y z xyz + + v t =
2 2
2
y z +

Ko mt tnh tng qut ta gi s x s y s z
Xt d = f(x, y, z) f(x, t, t) =
2
2( 2 ) ( ) y z t x yz t +
D thy 2 0 y z t + s v
2
0 yz t s nn khi 0 x s th 0 d s
Ta xt 2 trng hp:
Nu x s 0, khi f(x, y, z) s f(x, t, t). Ta s CM f(x, t, t) s 10
Tht vy, thay t =
2 2
2
y z +
=
2
9
2
x

f(x, t, t) =
2 2
1
2 2 2(9 ) (9 )
2
x x x x + = g(x)
Do 0 x s v
2
9 x s nn | | 3, 0 x e
Vy ta xt hm s g(x) =
2 2
1
2 2 2(9 ) (9 )
2
x x x x + / | | 3, 0
g(x) =
2
2
3 5 4
2 2
18 2
x x
x


C g(x) = 0 vi | | 3, 0 x e 1 x =
Ta c BBT



MT S PHNG PHP CHNG MINH BT NG THC
PHNG PHP DN BIN
Trang 22

x -3 -1 0
g(x) + 0
g(x)


-6
10
6

2
T BBT suy ra :
g(x) s 10 f(x, y, z) s f(x, t, t) = g(x) s 10 pcm
Du = xy ra khi x = -1, y = z =2.
Nu x > 0 y > 0, z > 0.
Khi ta s x l bi ton bng cch xt 2 trng hp
- Nu x>
3
4
th f(x, y, z)=
3
2 2 2
3 27
2( ) 2 3( ) 2 27 10
4 64
x y z xyz x y z
| |
+ + s + + = <
|
\ .

- Nu x <
3
4
th f(x, y, z)=
2 2
3 3
2( ) 2 2( 2 18 10
4 4
x y z xyz y z
| | | |
+ + s + + s + <
| |
\ . \ .

Vy BT c cm, du = xy ra khi chng hn 1 x = , 2 y z = =
V d B.II.4: [Mathlinks] Cho a, b, c l cc s dng c tch bng 1. CMR
1 1 1 6
5
a b c a b c
+ + + >
+ +

Chng minh
t f(a,b,c) =
1 1 1 6
5
a b c a b c
+ + +
+ +

Khng mt tnh tng qut ta c th gi s { } max , , a a b c =
Xt d = f(a,b,c) f(a, bc , bc )
=
( )
( )
2
1 6
( )
2
b c
bc
a b c a bc
(
(

(
+ + +


Do { } max , , a a b c = nn a bc > ; 2 b c bc + >

1 6 1 2 1
( )( 2 )) 9 1 0
3 3 ( )( 2 )
a b c a bc bc
bc bc bc a b c a bc
| |
+ + + > > = >
|
+ + + \ .
(1)
T (1) suy ra d > 0.
Vy f(a,b,c) > f(a,t,t) (vi 0 a t > > v
2
1 at = ) (*)
Xt f(a,t,t) =
2
2
3
1 2 6 2 6
5 5
2 1 2
t
t
a t a t t t
+ + = + +
+ +
(do
2
1
a
t
= )

( )
( )
( )
2
4 3 2
3
1
2 4 4 2
2 1
t
t t t t
t t

= + +
+
(2)
t g(t) =
4 3 2
2 4 4 2 t t t t + + / (0,1] ( do 0 a t > > v
2
1 at = nn 0 1 t < s )
C g(t) =
3 2
8 12 8 1 t t t +
'( ) 0 ( )( )( ) 0 g t t x t y t z = = (vi , 0, 0 1 y z x < < < )
MT S PHNG PHP CHNG MINH BT NG THC
PHNG PHP DN BIN
Trang 23

t = x
Ta c BBT
t 0 x 1
g(t) + 0 -
g(t)
g(x)
2 3
T BBT
0
( ) lim ( ) 2 ( ) 0 / (0;1]
t
g t g t g t

> = >
M
( )
( )
2
3
1
0
2 1
t
t t

>
+
( do t > 0) nn t (1) ( , , ) 0 f a t t > (**)
T (*) & (**) ( , , ) 0 f a b c > pcm (Du = xy ra khi a = b = c = 1)
V d B.II.5: Cho x, y, z 0 tha mn
2 2 2
3 x y z + + = . Chng minh rng:
( ) 7 12 9 xy yz zx xyz + + s +
Chng minh
t ( ) ( ) ( , , ) 12 9 7 12 9 7 7 f x y z xyz xy yz zx xyz xy z x y = + + + = + +
( ) ( ) ( )
( ) ( )
( )
( ) ( )
( )
( )
( )
2 2
2 2 2 2 2 2
2 2
2 2
2 2
2
2
2 2
2 2
3
2
( , , ) ( , , )
9 7 7 2 0
9 7 7 2
2
14
9 7
2( )
(9 7) 2 14
x y
t t t z x y z
f x y z f t t z
z xy t xy t z x y t
x y
xy z z x y x y
z x y
x y z
x y x y
z x y x y z
+
= + + = + + =
>
+ >
| | +
> + +
|
\ .

s
+ + +
+ + + s

Do
( )
2 2
2 x y x y + s + . Ta s chng minh
( ) ( ) ( )
2 2 2
7
9 7 2 7 9 2 3 7 z x y z z
z
| |
+ s s
|
\ .
(*)
t z = min{x,y,z} ta c 0 z 1
7
9 2
z
s v
( )
2
2 3 6 z s (*) ng.
Vy f(x, y, z) f( t, t, z). Ta s chng minh f(t, t, z) 0
2
3 2 z t = ta c
2 2 2 2
( , , ) 9 3 2 7 14 3 2 12 f t t z t t t t t = + vi
3
1,
2
t
(
e
(


t r = 1/t
2
ta phi chng minh
MT S PHNG PHP CHNG MINH BT NG THC
PHNG PHP DN BIN
Trang 24

2
( ) 9 3 7 14 3 2 12 0 g r r r
r
= + >
9 21 3 3
'( ) 12 7 12
3 2 3 2 3 2
g r
r r r r r r r
| |
= + = +
|
\ .
vi
2
,1
3
r
(
e
(


'( ) 0 1 g r r = = vi
2
,1
3
r
(
e
(

m
2 2
' 0 '( ) (1) 0 ,1
3 3
g g r g r
| | (
< s = e
|
(
\ .

Vy ( ) (1) 0 g r g > = . Ta c iu phi chng minh. Du = xy ra khi x = y = z = 1
Qua cc v d trn, ta thy vic kho st hm s l cng vic rt hay gp trong php
chng minh th 2 ca phng php dn bin. Vic nm vng nhng kin thc v o
hm, hm s v vy l rt quan trng.
Dn bin v gi tr trung bnh rt hu dng. Tuy nhin cc cch dn bin l v cng
phong ph v uyn chuyn. Nh ni trn, vic dng k thut chun ha trong
phng php dn bin cng rt hay gp. Chng ta s xt mt v d minh ha cho iu
ny
V d B.II.6: (Iran 1996) Chng minh rng vi a, b, c 0 th
( )
( ) ( ) ( )
2 2 2
1 1 1 9
4
ab bc ca
a b b c c a
| |
+ + + + > |
|
+ + +
\ .

Chng minh
y, du = xy ra ngoi a = b = c cn c , 0 a b c = =
V BT l ng bc nn ta s chun ha 1 ab bc ca + + = (*).
t
( ) ( ) ( )
2 2 2
1 1 1
( , , ) f a b c
a b b c c a
= + +
+ + +
. Ta cn chng minh
9
( , , )
4
f a b c >
Vi t tng ca dn bin, ta ngh n vic chng minh ( , , ) ( , , ) f a b c f t t c > . Do ( t, t, c)
cng tha mn (*) ta c
2
2 1 t tc + = .
( ) ( ) ( ) ( )
2 2 2 2 2
1 1 1 1 2
( , , ) ( , , )
4
d f a b c f t t c
t
a b b c c a t c
= = + +
+ + + +

Ta c ( ) ( )( )
2
2
2 t ct ab bc ca c t a c b c + = + + + = + + . T :
( ) ( ) ( ) ( ) ( ) ( ) ( )
( )
( )
( )
( )( )
( ) ( ) ( )
( ) ( )
( ) ( )
( )
( )
( )
( ) ( )
2
2
2
2 2 2
2
2 2
2 2 2
2
2
2 2 2
2 2
1 1 2
4
2
4
a b t a c b c c t a c b c a c b c
a b
a c b c
a c b c
t a b t a b
d
a b b c
b c t a c b c
a b t a b a b
a c b c
a c b c b c t
+ = + + + + = + + + + +

= + + =
+ + +
+ +
= + +
+ +
+ + +
+ +
= +
+ +
+ + + +

MT S PHNG PHP CHNG MINH BT NG THC
PHNG PHP DN BIN
Trang 25

( )
( ) ( )
( )
( )
2
2 2 2
2
2
1 2
4
t a b
a b
a c b c
a c b c b c t
| |
+ +
|
=
|
+ +
| + + + +
\ .

Gi s { } min , , c a b c = th
3
3
c s v
3
3
t >
( )
( )( ) ( ) ( ) ( ) ( )
2
2 , 2 t a b a c b c a c b c a c b c c t a b t + + s + + + + + = + + + s +
Do d 0, ta chng minh c ( , , ) ( , , ) f a b c f t t c >
Thay
2
1
2
t
c
t

= ta c
( )
( )
( ) ( )
( ) ( )
( )
( )( )
( )
2
2 4
2 4 2
2 2 2 2 2 2
2 2 2 2 2 2
2
2 4 6 6 4 2 2 2
2 2
2 2 2 2
1 32
1 2 1 8 33 2 1
( , , )
4 4
1 1 4 1 4 1
2
1 7 15 9 9 18 9 1 1 3
9 9
4 4
4 1 4 1
t t
t t t
f t t c
t t
t t t t t t
t
t
t t t t t t t t
t t t t
+ +
+ +
= + = + = =
| | + + +
+
|
\ .
+ + + +
= = + >
+ +

Bi ton c chng minh. Du bng xy ra khi a = b = c hoc a = b , c = 0 v cc hon v.
y l mt bi ton rt kh. Ngay c khi dng phng php dn bin, vic chng minh
( , , ) ( , , ) f a b c f t t c > v
9
( , , )
4
f t t c > u kh phc tp, i hi k thut bin i phi rt
thnh tho. y cng l mt v d in hnh th hin vic chn bin t thch hp vi iu
kin ca bi, cng nh kt hp chun ha v dn bin chng minh bt ng thc.
u im ca cch dn bin cng th hin rt r trong v d ny. Vi bt ng thc 3 bin
i xng cc tr t ti tm, c 3 hay 2 bin bng nhau, vic s dng dn bin t ra tht
hiu qu.
B.III. Dn bin bng k thut hm s
y l mt k thut rt quan trng ca phng php dn bin. Tuy nhin chng ti gii
thiu n ngay sau phn c bn nht l nhm trang b cho cc bn mt k thut cn thit trc
khi i qua cc mc sau. Hn na, chng ti ngh rng khi quen vi n th cc bn s khng
cn phi
phn bit cc tr t ti tm hay ti bin, v do mc tip theo s nh nhng hn.
Trong $2 chng ta thy rng chng t f(x, y, z) f(t, t, z) ta ch vic xt hiu d =
f(x, y, z) f(t, t, z) ri tm cch nh gi sao cho d 0. Tuy nhin, l v dng BT qu n
gin, ph hp vi cc bin i i s. Gi s ta phi lm vic vi biu thc f c dng, chng
hn, nh: f(x, y, z) = xk + yk + zk vi k > 0 th cc cch bin i i s s tr nn rt cng
knh v phc tp.
K thut hm s dng gii quyt cc trng hp nh vy. tng chnh th ny,
chng hn chng minh f(x, y, z) f(x, t, t) vi t = (y+z)/2, ta xt hm: g(s) = f(x, t+s, ts)
vi s 0. Sau chng minh g tng vi s 0 (thng thng dng cng c o hm rt tin
li), suy ra g(s) g(0), s 0, v ta s thu c iu mong mun. Mt trong nhng v d
quen thuc vi cc bn l dn bin bng hm li, tuy nhin di y chng ta s quan st k
thut dn bin trong bi cnh tng qut hn, cn vn v hm li s c tr li mt mc
sau trong bi ton vi n bin.
MT S PHNG PHP CHNG MINH BT NG THC
PHNG PHP DN BIN
Trang 26

Chng ti nhn mnh rng, y l mt k thut kh, bi n cha ng nhng nt rt tinh t
ca phng php dn bin. Nhng v d sau y th hin rt r v p v sc mnh ca
phng php dn bin.
V d B.III.1: Cho 0 k > v , , 0 a b c > .CMR:
3
min 2,
2
k k k
k
a b c
b c c a a b
| | | | | |
+ + >
`
| | |
+ + +
\ . \ . \ . )
(*)
Chng minh
Bc 1: Ta ch cn chng minh bt ng thc khi
3
2
2
k
=
ln3
1
ln 2
k = (!)
Tht vy
Nu
ln3 ln3 3 3
0 1 1 2 2 min 2,
ln 2 ln 2 2 2
k k
k k

s < + < < =
`
)

Do
ln3
0 1
ln 2
k s <
nn
ln3 ln3 ln3
1 1 1
ln 2 ln2 ln 2
k k k
a b c a b c
b c c a a b b c c a a b

| | | | | | | | | | | |
+ + > + +
| | | | | |
+ + + + + +
\ . \ . \ . \ . \ . \ .

Vy cn CM
ln3 ln3 ln3
1 1 1
ln 2 ln2 ln 2
2
a b c
b c c a a b

| | | | | |
+ + >
| | |
+ + +
\ . \ . \ .
hay nhn xt (!) ng.
Nu
ln3
1
ln 2
k > lm tng t .
Bc 2: Vi
ln3
1
ln 2
k = ,ta s cm BT ng.
Khng mt tnh tng qut c th gi s 1 a b c + + = v b c a > > .
t
2
b c
t
+
= v
2
b c
m

= suy ra , , 1 2 b t m c t m a t = + = = . Khi :
1 2
(*) ( ) 2
2 1 1
k k k
t t m t m
f m
t t m t m
+ | | | | | |
= + + >
| | |
+
\ . \ . \ .
vi
ln3
1
ln 2
k =
V c a > nn 3 1 0 t m > > , v 1 2 b c t > + = nn
1 1
2 3
t > >
Ta s kho st ( ) f m trn min | | 0, 3 1 m t e vi
1 1
,
3 2
t
(
e
(

l hng s.
Ta c
1 1 1 1
1 1 1 1
( ) ( ) ( ) ( )
'( ) 0
(1 ) (1 ) (1 ) (1 )
k k k k
k k k k
k t m k t m k t m k t m
f m
t m m t t m m t

+ + + +
+ +
= > >
+ +

| | | |
1
( ) ln( ) ln( ) ln(1 ) ln(1 ) 0
1
k
g m t m t m t m t m
k
+
= + + >


Ta c
1 1 1 1 1
'( ) 0
1 1 1
k
g m
t m t m k t m t m
+ | | | |
= + + + >
| |
+ +
\ . \ .

(1)

2 2 2 2
2 1 2(1 ) 1 1
0 0
( )( ) 1 (1 )(1 ) 1 (1 )
t k t t k t
t m t m k t m t m t m k t m
+ +
+ > + >
+ +
MT S PHNG PHP CHNG MINH BT NG THC
PHNG PHP DN BIN
Trang 27


Do
ln3
1
ln 2
k = nn
1
2
1
k
k
+
>

suy ra chng minh (1) ta cn phi chng minh


2 3 2 2
2 2 2 2
2(1 )
0 ( ) 4 3 3 2 0
(1 )
t t
u m t t t tm m
t m t m

+ > = + >


Ta c
2
1 1
'( ) 2(3 2) 0 , ( ) (3 1) 2(3 1)(2 1) 0
3 2
u m t m t u m u t t t
(
= < e > = >
(


(1) ng '( ) 0 ( ) g m g m > ng bin ( ) (0) 0 '( ) 0 g m g f m > = >
( ) f m ng bin ( ) (0) f m f > =
Bc 3: Ta cn chng minh
1 2 1
(0) ( ) 2 2, 0,
2 1 3
k k
t t
f h t t
t t
| | | | (
= = + > e
| |
(

\ . \ .

| |
1 1
1
1 2
1 1
2 (1 2 )
'( ) 0 2 (1 )(1 2 )
(1 ) 2
k k
k
k k
k k k
kt k t
h t t t t
t t

+
+ +

= s s

(2)
Trong bt ng thc cui, v tri l hm ng bin theo t v v phi l hm nghch bin theo
t, v lu la
1
3
t s nn chng minh (2) ta cn chng minh:
1 2
1
1 1 2
2 1 1
3 3 3
k k
k

+
( | | | || |
s
| | | (
\ . \ .\ .
. Bt ng thc ny ng nn ( ) h t nghch bin, suy ra
1
( ) 2.
3
h t h
| |
> =
|
\ .
Vy bi ton c chng minh.
Nhn xt: thy c nt p ca bi ton ny, ta xt cc trng hp c bit.
Trng hp 1 k = , ta thu c BT Nesbit:
3
2
cyc
a
b c
>
+


Trng hp
1
2
k = , ta c Bt thc sau:
2
cyc
a
b c
>
+


y l 1 bi ton kh th v vi 1 li gii n gin s dng BT thc AM - GM:
2 2( )
2
2 ( )
cyc cyc
a a a b c
b c a b c a b c
+ +
= = =
+ + + +


V d B.III.2: (Phm Kim Hng) Cho cc s thc dng a, b, c c tch bng 1. Chng minh
rng:
a)
( )( )( ) ( )
4
2 2 2
81 1 1 1 8 a b c a b c + + + s + +
b)
( )( )( ) ( )
6
3 3 3
64 1 1 1 a b c a b c + + + s + +
Chng minh
a) t ( ) ( ) ( )( )
4
2 2 2
( , , ) 8 81 1 1 1 f a b c a b c a b c = + + + + + . Gi s a b c ta c c 1.
Xt hm s
MT S PHNG PHP CHNG MINH BT NG THC
PHNG PHP DN BIN
Trang 28

( )( )
( )
4
2
2 2 2
2
4
2
2 2 2 2 2
2
( ) ( , , ) 8 81 1 1 1
8 81 1 1
b b b
g t f ta c ta c c a t
t t t
b b
ta c c a b a t
t t
| |
| |
= = + + + + +
| |
\ .
\ .
| |
| |
= + + + + + +
| |
\ .
\ .

vi ,1
b
t
a
(
e
(

ta c:
( )
( )
3
2
2 2
2 3
3
2
2 2
2
'( ) 32 81 2 1
32 162 1
b b b
g t a ta c a t c
t t t
b b b b
a ta c a ta c
t t t t
| |
| | | |
= + + +
| | |
\ . \ .
\ .
| || | | || |
= + + + +
| | | |
\ .\ . \ .\ .

t
b
d ta
t
= + , d thy c 1 nn
1
2
d
c
c
s s . Vy
( ) ( ) ( ) ( )
3
2 2
3
32 32.9 32.9. 432 216 1 162 1
2
d c d c d d d c d c + > + > = > + > + do c 1
Do
2
,1 0
b b
t a
a t
(
e >
(

. Vy '( ) 0 g t > g(t) ng bin trn ,1
b
a
(
(


(1) ( , , ) ( , , )
b
g g f a b c f s s c
a
| |
> >
|
|
\ .
vi s ab =
Thay
1
s
c
= ta c
( )
4 2
2
2
9 9 5 3
5 4 3 2
2 2 2 2
2 1
( , , ) 8 81 1 1
1
8 16 24 96 87 78 99 120 21 94 47 0
f s s c c c
c c
c
c c c c c c c c c c
c
| | | |
= + + +
| |
\ . \ .
| | | |
= + + + + + + + + + >
| |
|
\ . \ .

Bt ng thc c chng minh. Du = xy ra khi a = b = c = 1
b) t ( ) ( )( )( )
6
3 3 3
( , , ) 64 1 1 1 f a b c a b c a b c = + + + + + . Gi s a b c nn c 1.
Xt hm s ( ) ( , , )
b
g t f at c
t
= vi ,1
b
t
a
(
e
(


( ) ( ) ( )
6 6
3 3
3 3 3 3 3 3 3 3
3 3
( ) 64 1 1 1 64 1 1
b b b b
g t at c a t c at c a t a b c
t t t t
| | | |
| | | |
= + + + + + = + + + + + +
| | | |
\ . \ .
\ . \ .
( )
( )
5
3 2
3 2 3
2 6
5
2
2 2 3
2 2 2
3
'( ) 6 64 3 1
6 192 1
b b b t
g t a ta c a t c
t t t
b b b b
a ta c a a t ab c
t t t t
| |
| || |
= + + +
| | |
\ .\ .
\ .
| |
| || | | |
= + + + + +
| | | |
\ .\ . \ .
\ .

MT S PHNG PHP CHNG MINH BT NG THC
PHNG PHP DN BIN
Trang 29

V ,1
b
t
a
(
e
(

nn g(t) 0 nu
( )
5
2
2 2 3
2
32 1
b b
ta c t a ab c
t t
| |
| |
+ + > + + +
| |
\ .
\ .

t
2
2 2
2
b
d t a ab
t
= + + , ta c:
5 4
3
b b
ta c ta c
t t
| | | |
+ + > + +
| |
\ . \ .

( ) ( ) ( )
2
2
2 2
3 2 2 2 3 6 72 36 1 32 1
bc
d ab tac d d d c d c
t
| |
= + + + > + > > + > +
|
\ .
v c 1
Vy g(t) 0 hay g(t) ng bin trn ,1
b
a
(
(

, do (1)
b
g g
a
| |
>
|
|
\ .
tc l
( , , ) ( , , ) f a b c f s s c > vi s ab = .
Thay
1
s
c
= , ta c:
( )
6 2
3
6 4 3
2 1
( , , ) 64 1 1
64
12 4 32 112 0
f s s c c c
c c c
c c c c c c
c c
| | | |
= + + +
| |
\ . \ .
= + + + + >

(do c 1). Ta c iu phi chng minh
y l mt v d v cch dn bin v trung bnh nhn. C th thy vic bin i, nh gi
cng nh kho st hm s l kh phc tp. ngh ra nhng li gii nh vy cn c mt
s kin tr rn luyn. V vy iu quan trng nht ca chng ta l phi c thc lm n
ni n chn, khng c b d gia chng, d phi i mt vi nhng bin i rt
khng khip. Khi gii c bi ton chng ta s cm thy mnh nhn c thnh qu
xng ng.
Tip theo l mt bt ng thc m kh n gin, nhng vic gii n th khng d cht no.
Phng php dn bin l mt phng php hiu qu gii bi ton ny.
V d B.III.3: Cho
, , 0
3
a b c
a b c
>

+ + =

. Tm gi tr ln nht ca
2 2 2
3 3 3
ab bc ca
S
c a b
= + +
+ + +

Chng minh
Khng mt tng qut a b c.
t a = s + t, b = s t th
( ) ( )
2 2
2 2 2
( ) ( )
( )
3
3 3
c s t c s t s t
S f t
c
s t s t
+
= = + +
+
+ + +

Kho st f(t) trn | | 0, t s c e
( )
( )
( )
( )
( )
( )
( )( )
| | ( ) ( )
2 2 2 2
2 2 2 2 2
2 2
2 2
2 2
2 2 2
2 2
2
'( )
3
3 3
3 3
8
4 2
, 0, , 3 , 3
3
c s t c s t
c c t
f t
c
s t s t
s t s t
cst s t u v
cst t
t s c u s t v s t
uv u v c

= + +
+
+ + + ( (
+ + +

+
= + e = + + = +
+

MT S PHNG PHP CHNG MINH BT NG THC
PHNG PHP DN BIN
Trang 30

C 2 2 s a b = + > , vy
3
1,
2
s
(
e
(

nn (3 2 ) 1 cs s s = s . Mt khc ( )
2
3 4, u s t = + + >
( )
2
2
3 3 v s t c = + > + do | |
2
4
0, (*)
3
cst t
t s c
uv c
e s
+

Theo bt ng thc Cauchy:
( ) ( ) ( )
2 2
2 2
2 2 2 2 2
3 3 16 u v s t s t s t
( (
= + + + >

(1)
V ( ) ( ) ( )( )
3
2 2 2
2 2 2 2
4 3 3
8 3 16 3 3 4
3
cs s t c
cs u v c cs s t c
| | + + + + +
+ + = + + + s
|
\ .

Thay c = 3 2s v t s c = 3s 3 ta c:
( ) ( ) ( )
( )( )
2 2
2 2 2 2
2 2 2 2 2
4 3 3 4 3 2 6 3 3 3 2
12 8 6 9 18 9 9 12 4 36 30 6 6 2 3 12
cs s t c s s s s s
s s s s s s s s s s
+ + + + + s + + + +
= + + + + + + = + = s

Do
3
1,
2
s
(
e
(

. T suy ra ( )( )
2 4
8 3 4 cs u v c + + s (2)
T (1) v (2) ta c
( )( )
2 2
2 2 2
8
(**)
3
cst s t u v
t
u v c
+
s
+

T (*) v (**) suy ra f (t) 0 vi | | 0, t s c e
Vy
( )
2 2
2 2 2 2
2 2 (3 2 )
( ) (0) ( )
3 3 3
3 3 2
cs s s s s
f t f g s
s c s
s

s = + = + =
+ + +
+
(1)
Kho st g(s) trn
3
1,
2
s
(
e
(

. Ta c
( )( ) ( )
( )
( ) ( )
( )
( )
( )
( )( ) ( )
( ) ( )
2
2
2 2
2 2
2 2
2
2 2
2 2
2 2 2
2
2 2 2
2
2 3 3 2 8 12
6 8 3 2 6 4
'( )
3
3 3 2
108 3 4 1 3 6
6 24 18 12 24
3
3 3 2 3 3 3 2
s s s s
s s s s s
g s
s
s
s s s s s
s s s s
s
s s s
(
+
+

= +
( +
+

+ +
+ +
= + =
( ( +
+ + +


D thy s
2
3s + 4 > 0 v s
2
-3s + 6 =
33 3 33 3
2 3
s s
| || |
+
+
| |
| |
\ .\ .
nn g (s) dng trn
(1,s
0
) v m trn (s
0
,3/2) vi
0
33 3
2
s

=
Vy
3
1,
2
s
(
e
(

th ( )
0
11 33 45
( )
24
g s g s

s = (2)
Vy gi tr ln nht ca S l
11 33 45
24

. Gi tr t c khi xy ra ng thc (1) v (2)


tc l t = 0 v s = s
0,
tc l a = b = s
0
v c = 3 2s
0
hay a = b =
33 3
2

, c = 6 33 v cc
hon v
MT S PHNG PHP CHNG MINH BT NG THC
PHNG PHP DN BIN
Trang 31

B.IV. Bt ng thc 3 bin cc tr t ti bin
Nu nh trong phn trc chng ta c th hiu "dn bin" l "y hai bin li gn nhau", th
trong trng hp ny ta phi hiu "dn bin" ngha l "y 1 bin ra bin". Chng hn nh
xt BT f(x, y, z) 0 vi x, y, z 0, ta c th hi vng vo nh gi f(x, y, z) f(0, s, t), trong
s, t l cc i lng thch hp sinh ra t cc bin a, b, c (ta s gi y l k thut dn 1
bin ra bin). Tt nhin ta s chn s, t sao cho hiu d = f(x, y, z) f(0, s, t) l n gin v c
th nh gi thun li. Cui cng ta ch vic kim chng f(0, s, t) 0.
T l thuyt nh vy n thc hnh cng l c mt vn . Chng ta s xt mt s v
d rt ra nhng kinh nghim cho mnh v vic dn bin ra bin
V d B.IV.1: Cho a, b, c 0, ab + bc + ac = 1 (*). Chng minh rng
1 1 1 1
2
2 a b c b a c
+ + > +
+ + +

Chng minh
D thy ng thc khng xy ra ti tm (a = b = c =
1
3
). Ta ngh n vic mt bin bng 0.
Ta s chng minh ( , , ) (0, , ) f a b c f s t > vi s, t thch hp. Ly
1
, s a b t
a b
= + =
+
tha mn (*)
Vic xt hiu y kh kh khn. Ta biu din ( , , ) f a b c v cc bn x = a + b v c nh sau:
( )( )
( )( )
( )( )
2
2
1 1 1 1
( , , )
2
1 1 2 2 1
( )
1
b c c a
f a b c
a b c b a c a b b c c a
c a b c b c c a
c x c
g c
a b x b c c a c
+ + +
= + + = +
+ + + + + +
+ + + + + +
+ + +
= + = + =
+ + + +

Ta c:
( )
( )
( )
( )
( )
( )
2
2 2
2
2
2
2 2 2
2 2
3 3
2 2 2 2
2
2
1
1 2 2 1 .
1
2 2 2 1
'( )
1
1 1 2 2 1
1 1
1 2 2 1 1 2 2 1
c
c
c
c c x c
c
c x c
g c
c
c c c c x c
c cx c c
c c x c c c x c
+
+
+ + + +
+
+ +
=
+
+ + + + + +
+
= =
+ + + + + + + +

Gi s c = max {a, b, c} ta c
2 2
1 0 c cx ab c = s . Vy g (c) 0. Do g (c) nghch
bin. Li c cx = ca + ab ab + bc + ca 1 hay
1
c
x
s .
Do
MT S PHNG PHP CHNG MINH BT NG THC
PHNG PHP DN BIN
Trang 32

( )
2
2
2
3 2
2 2
2
2
2
2 1
2
1 1
1 1 1 2 2 1 1
( ) .
( 1) 1
1
1
( , , ) (0, , )
1
x
x
x
x x x x
x x
g c g x
x x x x x x
x
x
x
x f a b c f s t
x x
+
+ +
+ +
+ + + | |
> = + = + = +
|
+ +
\ . +
= + + >
+

Ta cn phi chng minh
2
1 1
( ) 2
1 2
x
h x x
x x
= + + > +
+

( )
( )
( )
2
2 2
2
2
3 3
2 2
1
1 1 1 1 2 1 1 2
1
'( ) 1
1 2 2 2 2
2 1 2 1
x x
x
x x x x x
x
h x x
x x x x x x x x
x x x x
+
| |
+ + | +
= + + = + =
|
+
| + +
\ .
( )
( ) ( )
( )
( )
( ) ( )
( ) ( ) ( )
( ) ( )
( ) ( ) ( )
3 3 2 2 2 2 2 2
3 3 3
2 2 2 2 2
3
4 3 2
3 3
2
2 2 2
1 2 1 1 2 1
1 1
2 1 2 1 1 2 1
1 2 4 2 1
2 1 1 2 1
x x x x x x
x x
x x x x x x x x x
x x x x x
x x x x x x
+ + + +
= =
| |
+ + + + +
|
\ .
+ + + +
=
| |
+ + + +
|
\ .

Do x 0 nn h(x) dng khi x > 1 v m khi x < 1. T ta thy h(x) h(1) =
1
2
2
+ vi
x 0.
Bt ng thc c chng minh. Du bng xy ra khi c = 0 a = b = 1 v cc hon v.
Vic dn bin ra bin y rt tinh t, th hin c vic chn s, t v vic bin i f (a, b,
c) thnh g(c) nh lm. Chng ta c th thy vic quy t bt ng thc 3 bin v 2 bin
v ri 1 bin trong chng minh trn. Trong cc bc chng minh, vic dng o hm l
ht sc quan trng.
Ni ring v vic bin i bt ng thc t 3 bin thnh 2 bin, y l mt k thut kh
kh v tinh t, i hi mt s thnh tho v k nng bin i. Tuy nhin, nu thnh tho
th y cng l mt cng c ht sc hu dng chng minh bt ng thc bng phng
php bin i. Vic bin i nhiu khi cng tht phc tp. Hy vng chng ta s c kin
tr rn luyn k thut ny.
V d B.IV.2: (Jack Grafukel) Cho , , 0. a b c > Chng minh rng:
5
4
a b c
a b c
a b b c c a
+ + s + +
+ + +
(1)
Nhn xt
Ta xt xem du = xy ra khi no.
Do a b c = = khng tho mn nn mt cch t nhin ta ngh n trng hp bin l 0 c = , khi
bt ng thc (1) tr thnh
5
4
a
b a b
a b
+ s +
+
(2)
MT S PHNG PHP CHNG MINH BT NG THC
PHNG PHP DN BIN
Trang 33

Chun ho 1 a b + = . Ta c (1)
2
5 1
1 0
4 2
b b b
| |
+ s >
|
\ .
( lun ng)
V bt ng thc xy ra khi 3 0, 0 a b c = > = (v cc hon v).
Do du bng xy ra khi c 3 bin ri nhau nn khn th thc hin phng php dn bin v 2
bin bng nhau. Ta s thc hin theo phng php dn mt bin v bin.
Chng minh
Khng mt tnh tng qut ta c th gi s { } ax , , a m a b c = , chun ha cho 1 a b c + + = .
t
2
a c
t
+
= v
2
a c
s

= , th , , 1 2 a t s c t s b t = + = =
Khi bt ng thc (1) ( )
1 2 5
4 1 1 2
t s t t s
f s
s t t s t
+
= + + s
+
(3)
Ta s chng minh ( ) ( ) ( ) { }
ax 0 , 1 f s m f f s vi | | 0; s t e
Ta c ( )
( ) ( )
3 3
2 2
1 1 2 1
'
1 2
2 1 2 1
t s t
f s
s t t
s t t s
+
= + +
+
+

V cha xc nh c du ca s bn ta o hm tip
( )
( )
( )
( )
( )
( )
3 5 5
2 2 2
3 3 1 2
1
''
1 4 1 4 1
t s t
f s
s t s t t s
+
= + +
+ +
.S dng 1 2 0 b t = > suy ra
( )
( )
( )
( )
( )
( ) ( )
( )
( )
5 7 7 7 7
2 2 2 2 2
15 15 1 2 15 1 2
9 18 3 33
''' 0
4 1 8 1 8 1 8 1 8 1
t s t t
s t
f s
s t s t t s s t t s
+
+
= + = + >
+ + +

Do ( ) | | ''' 0 0; f s s t > e nn theo nh l Rolle th phng trnh ( ) ' 0 f s = c ti a 2 nghim
trn | | 0; t . Mt khc d dng chng minh ( ) ' 0 0 f s v ( ) ' 0 f t > , nn ( ) ' f s ch c th i
du ti a 1 ln trn ( ) 0; t , v hn na ( ) ' f s ch c th c mt trong cc dng sau:
( ) ( ) ' 0, 0; f s s t > e hoc ( ) ( ) ' 0, 0; f s s t < e Hoc ( ) ' f s c dng 0 + trn ( ) 0; t . Tuy
nhin trong c 3 trng hp th ( ) f s cng ch c th t cc tr ti bin. T suy ra
( ) ( ) ( ) { }
ax 0 , 1 f s m f f s vi | | 0; s t e
D dng chng minh ( )
5
0
4
f s v ( )
5
4
f t s nn ( ) ( ) ( ) { }
5
ax 0 , 1
4
f s m f f s s
Ch : Bng cch tng t ta c th chng minh bi ton tng qut sau:
Cho , , 0 a b c > v ( ) 0;1 k e Khi ta c BT:
( ) ( ) ( )
( )
1 k
k k k k
a b c
C a b c
a b b c c a

+ + s + +
+ + +
Vi
2
1
k
C k k = +
Trong trng hp 3 bin, thng thng chng ta c nh 1 bin v thay th 2 bin cn li.
Tuy nhin, i khi chng ta c th lm khc hn bng cch ch thay th 1 bin hoc thay
th c 3 bin. Sau y l mt v d v phng php dn bin ra bin trong bt ng thc
MT S PHNG PHP CHNG MINH BT NG THC
PHNG PHP DN BIN
Trang 34

hon v, cng l v d minh ha vic c nh 2 bin v thay th 1 bin. Li gii ca bi
ton kh n gin, gip ta hiu r hn v t tng c bn ca dn bin ra bin.
V d B.IV.3: Cho a, b, c 0, chng minh rng
( ) ( )( )( )( )
2
2 2 2
4 a b c a b c a b b c c a + + > + +
Chng minh
t
( ) ( )( )( ) ( )
2
2 2 2
( , , ) 4 f a b c a b c a b c a b b c c a = + + + + Khng mt tng qut, gi
s c = min{a, b, c}. Nu a b c th VT 0 BT hin nhin ng. V vy ta ch cn xt b
a c.
( ) ( ) ( ) ( )( ) ( ) ( )
( ) ( ) ( )( ) ( )
( ) ( ) ( ) ( ) ( )
( ) ( ) ( ) ( )
2 2
2 2 2 2 2
2
2 2 2 2 2 2 2
2
2 2 2 2 2 2 2 2
2
2 2 2 2 2 2
( , , ) ( , , 0) 4
( ) 4
( ) 4
( ) 4
f a b c f a b a b c a b a b a b c b c c a a b ab
a b c a b b a a b c bc ab c ac ab a b
a b c a b b a a b bc c ac c bc ab c ac
a b c a b b a c a b a b c bc a
= + + + + + + + (

(
= + + + + + + + + +

(
= + + + + + + + +

= + + + + + + +
( ) ( ) ( )
2
2
2 2 2 2 2 2 2 2 2
( ) 4
0
b c ac
a b c a b b a c a b ab c
( +

= + + + + + +
>
M
( ) ( ) ( )
2 2
2 2 2 2 2 2
( , , 0) 4 2 0 f a b a b b a ab a ab b = + = + >
Vy ta chng minh xong. Du bng xy ra khi
2 2
0, 2 0 c a ab b = + = hay
( )
0, 3 1 c a b = = v cc hon v.
B.V. nh l dn bin mnh S.M.V v bt ng thc 4
bin
nh l S.M.V
B Gi s a
1
, a
2
, , a
n
l dy s thc ty . Ta thc hin lin tip php bin i
sau
1. Chn i, j { } 1, 2,..., n e l 2 ch s sao cho
a
i
= min {a
1
, a
2
, , a
n
} a
j
= max {a
1
, a
2
, , a
n
}

2. Thay a
i
, a
j
bi
2
i j
a a +
(nhng vn gi ng th t ca chng trong dy s)
Khi sau v hn ln thc hin bin i ni trn th mi s a
i
u tin ti gii hn
1 2
...
n
a a a
a
n
+ + +
=
CHNG MINH
Php bin i ni trn, t gi tr i ta s gi l php bin i . K hiu dy ban u l
1 1 1
1 2
, ,...,
n
a a a . Sau mt php bin i ta c dy mi k hiu l
( )
2 2 2
1 2
, ,...,
n
a a a . Lm tng t,
t dy
( )
1 2
, ,...,
k k k
n
a a a ta thu c dy mi k hiu l
( )
1 1 1
1 2
, ,...,
k k k
n
a a a
+ + +
. Khi vi mi s
nguyn 1, i n = ta phi chng minh
MT S PHNG PHP CHNG MINH BT NG THC
PHNG PHP DN BIN
Trang 35

1 2
...
lim ,
k n
i
k
a a a
a a a
n

+ + +
= =
Ta t
( )
1 2
min , ,...,
k k k
k n
m a a a = v
( )
1 2
max , ,...,
k k k
k n
M a a a =
D thy php bin i khng lm tng gi tr ca M
k
v khng lm gim gi tr ca m
k
. V
m
k
v M
k
u l cc dy b chn nn tn ti
lim
k
k
m m

= lim
k
k
M M

=
Ta phi chng minh m = M. Phn chng gi s M > m, t d
k
= M
k
m
k
. Ta c mt nhn xt
n gin sau y
Nhn xt. Gi s sau mt s php bin i dy
1 1 1
1 2
, ,...,
n
a a a tr thnh dy
( )
1 2
, ,...,
k k k
n
a a a
sao cho
1 1
2
k
M m
m
+
= th ta c
1 1
2
2
M m
m
+
=
Tht vy, khng mt tnh tng qut ca bi ton ta gi s
1 1 1
1 1 2 1
...
n
M a a a m = > > > = . cho
gn, ta s k hiu
i
a thay cho
1
i
a .
Nu
1
2
n
k
a a
m
+
= v k l ch s nh nht tha mn th
2
1,
i k
a m i n > = . iu ny suy ra trc
tip t tnh cht khng gim ca m
i
v ch rng
1 2
2
a a +
l mt s hng no ca
dy
( )
2 2 2
1 2
, ,...,
n
a a a .
T nhn xt trn ta thu c mt kt qu quan trng hn. t
{ } { }
{ } { }
: | 2 | 2
: | 2 | 2
k k l k k k l
k k l k k k l
S k l k m M m S k m M m
P k l k m M M P k m M M
+
+
= - > + = = + =
= - > + = = + =

Nu S hoc P c v hn phn t, gi s |S| = th, vi mi k S
1 1 1 1
2 2 2
k k k k k
k k k k
m M M m d
d M m M
+ + + +
+
= = s =
V d
r
l dy gim nn nu |S| = th lim 0
r
r
d

= v do M = m.
Nu khng, th |S|, |P| < +. Ta c th gi s |S| = |P| = 0 m khng lm nh hng n kt
qu ca bi ton. Khi vi mi k > 1 th s
1
2
n
a a +
khng th l s nh nht hay ln nht
trong dy
( )
1 2
, ,...,
k k k
n
a a a v nh vy ta c th xt bi ton p hn vi n 1 s, sau khi b
i mt s
1
2
n
a a +
. Bng phng php quy np n gin ta thu c pcm.
T b trn y ta suy ra mt kt qu trc tip
NH L DN BIN MNH S.M.V Nu : ,
k
f I R R I c = [, ] x [, ] x x
[, ],, R l hm l i n t c i xng v b chn di tha mn iu kin
1 2 1 2
( , ,..., ) ( , ,..., )
n n
f a a a f b b b >
Vi
1 2
( , ,..., )
n
b b b l dy thu c t dy
1 2
( , ,..., )
n
a a a theo php bin i , th ta c
1 2
( , ,..., ) ( , ,..., )
n
f a a a f a a a > vi
1 2
...
n
a a a
a
n
+ + +
= .
MT S PHNG PHP CHNG MINH BT NG THC
PHNG PHP DN BIN
Trang 36

Bng nh l ny, khi s dng dn bin ta ch cn chn ra s nh nht v s ln nht.
nh l v dn bin c chng minh kh cht ch v c mt kt qu mnh hn hon ton
bng kin thc s cp, chng ta hon ton c th p dng c.
Ngoi ra php bin i c th khc hn, chng hn thay thnh
2 2
,
2
a b
ab
+
hoc
bt k mt dng trung bnh no khc. Ty theo gi thit ca bi ton m ta cn chn cch dn
bin cho ph hp.
V d B.V.1: [IMOSL, Vit Nam ngh] Cho , , , 0, 1 a b c d a b c d > + + + =
Chng minh rng:
1 176
27 27
abc bcd cda dab abcd + + + s + (1)
Chng minh
ng thc xy ra khi
1
4
a b c d = = = = hoc
1
, 0
3
a b c d = = = = ( v cc hon v ).
Do , nhng nh gi thng thng rt d ri vo b tc.
t ( )
176
, , ,
27
f a b c d abc bcd cda dab abcd = + + +
Ta c:
( ) ( )
176
, , ,
27
f a b c d ab c d cd cd a b
| |
= + + +
|
\ .

t ( ) ( )
2 1 176
, , , , ,
2 2 2 27
a b a b
k f a b c d f c d a b c d cd
+ + | | | |
= = +
| |
\ . \ .

Nu
176
0 0 ( , , , ) , , ,
27 2 2
a b a b
c d cd k f a b c d f c d
+ + | |
+ > s s
|
\ .

Khi theo nh l dn bin mnh ta c
1
( , , , ) , , ,
4 4 4 4 27
a b c d a b c d a b c d a b c d
f a b c d f
+ + + + + + + + + + + + | |
s =
|
\ .
pcm.
Nu ( ) ( )
( )
3
176 1
0 , , ,
27 3 27
c d a b
c d cd f a b c d cd a b
( + + +
+ s s + s =
(

pcm.
Trn y l mt v d kh in hnh v th hin r u im ca phng php dn bin
mnh. Phng php ny hu dng nht khi s dng vi bt ng thc 4 bin. Chng ta
s xt mt v d s dng nh l S.M.V bng cch thay s ln nht v 1 s bng trung
bnh cng ca chng.
V d B.V.2: Cho , , , 0 a b c d > v 4 a b c d + + + = Chng minh rng:
( ) ( ) ( ) ( )
2 2 2 2
8 abc bcd cda dab abc bcd cda dab + + + + + + + s
Chng minh
t ( ) ( ) ( ) ( ) ( )
2 2 2 2
, , , f a b c d abc bcd cda dab abc bcd cda dab = + + + + + + +
Gi s a b c d > > > .Xt hiu: ( ) , , , , , ,
2 2
a c a c
f b d f a b c d
+ + | |

|
\ .

MT S PHNG PHP CHNG MINH BT NG THC
PHNG PHP DN BIN
Trang 37


( ) ( )
( )
2 2
2 2 2 2
2
2 2
2
2 2
4 2 0
2
a c a c
b d ac b d b d
a b
b d abcd b d
(
+ | | | |
= + + + +
( `
| |
\ . \ .
(
)
| |
> + + >
|
\ .

(do
2 2
abcd b d > )
V vy ( ) , , , , , , .
2 2
a c a c
f a b c d f b d
+ + | |
s
|
\ .
Theo nh l , ta ch cn chng minh bt ng thc
vi
4
, 4 3 , 0
3
a b c x d x x = = = = s s
D dng chng minh:
4 3 2
28 16 12 8 0 x x x s vi
4
0
3
x s s (S dng o hm)
Bt ng thc c chng minh. ng thc xy ra khi v ch khi 1 a b c d = = = =
V d B.V.3: [Bt ng thc Tukervic] Cho , , , 0 a b c d > . Chng minh rng:
4 4 4 4 2 2 2 2 2 2 2 2 2 2 2 2
2 a b c d abcd a b b c c d d a a c b d + + + + > + + + + +
Chng minh
Gi s a b c d > > > . Xt :
( )
4 4 4 4 2 2 2 2 2 2 2 2 2 2 2 2
, , , 2 f a b c d a b c d abcd a b b c c d d a a c b d = + + + +
( ) ( ) ( )
4 4 4 4 2 2 2 2 2 2 2 2
, , , 2 f a b c d a b c d abcd a c b d a c b d = = + + + + + +
( )
( )
( ) ( )
2 2
2 2
, , , , , , 0 f a b c d f ac b ac d a c a c b d
(
= + >


Do theo nh l S.M.V, ta ch cn chng minh bt ng thc khi a b c t = = =
Khi Bt ng thc cho
4 4 3 4 2 2 4 3 3 2 2
3 2 3 3 3 t d t d t t d d t d t d t d + + > + + + >
Hin nhin ng theo bt ng thc AM-GM.
ng thc xy ra a b c d = = = hoc , 0 a b c d = = = v cc hon v.
V d B.V.4: Cho , , , 0 a b c d > v 4 a b c d + + + = .Chng minh rng:
( ) ( ) ( ) ( ) ( ) ( ) ( ) ( )
2 2 2 2
1 1 1 1 1 1 1 1 a b c d a b c d + + + + > + + + +
Chng minh
t ( ) ( ) ( ) ( ) ( ) ( ) ( ) ( ) ( )
2 2 2 2
, , , 1 1 1 1 1 1 1 1 f a b c d a b c d a b c d = + + + + + + + +
V gi s a b c d s s s .Xt hiu: ( ) , , , , , ,
2 2
a c a c
P f a b c d f b d
+ + | |
=
|
\ .

Ta s chng minh 0 P > .Tht vy, d thy 2 a c + s nn
( ) ( ) ( )
( )
2
2
2
2
2 2
4
1
1 1 1 0
2 2 16
a c ac
a c
a c a c
| |
(
+ +
+ | |
| + + + = >
(
|
|
\ .
(

\ .

M theo bt ng thc AM-GM th: ( ) ( )
2
1 1 1
2
a c
a c
+ | |
+ + s +
|
\ .
.
MT S PHNG PHP CHNG MINH BT NG THC
PHNG PHP DN BIN
Trang 38

Do : ( ) , , , , , ,
2 2
a c a c
f a b c d f b d
+ + | |
>
|
\ .

Vy ta ch cn chng minh bt ng thc vi , 4 3 a b d x c x = = = = :
Ta c ( ) ( ) ( ) ( ) ( )
3
2 3
2
, , , 4 3 1 1 4 3 1 5 3 f x x x x x x x x
(
= + + +


( ) ( ) ( ) ( )
6 4 2 2 3 2
3 3 1 9 24 17 3 3 1 5 3 x x x x x x x x x = + + + + + + +
8 7 6 5 4 3 2
9 24 44 72 81 68 54 36 12 x x x x x x x x = + + + +
( ) ( )
2
6 5 4 3 2
9 6 23 2 18 12 12 1 x x x x x x x = + + +
( ) ( ) ( ) ( )
2 2 2 2
4 4 2 2
3 1 2 5 2 1 10 3 2 1 0 x x x x x x x x
(
+ + + + >


Vy Bt ng thc c chng minh. ng thc xy ra khi v ch khi 1 a b c d = = = =
V d B.V.5: Cho , , , 0 x y z t > tho mn 4 x y z t + + + = . Chng minh rng:
( ) ( ) ( ) ( ) 1 3 1 3 1 3 1 3 125 131 x y z t xyzt + + + + s +
Chng minh
t: ( ) ( ) ( ) ( ) ( ) , , , 1 3 1 3 1 3 1 3 131 f x y z t x y z t xyzt = + + + +
Khng mt tnh tng qut ta gi s x y z t > > > . Khi :
V x y z t > > > nn 2 y t + s . Do d thy: ( ) ( ) 9 1 3 1 3 131 y t yt + + >
Vy ta c ( ) , , , , , ,
2 2
x z x z
f x y z t f y t
+ + | |
s
|
\ .
. Theo nh l S.M.V th bt ng thc s c
chng minh xong nu 1 4 3 x y z a t a = = = > > =
Tht vy, khi cn chng minh
( ) ( ) ( ) ( ) ( ) ( )
2 2
3
1 3 1 3 4 3 125 131 4 3 1 3 4 50 28 0 a a a a a a a ( + + s + + s


D thy bt ng thc nu hin nhin ng.
Vy BT cho c chng minh. Du = xy ra khi 1 x y z t = = = =
hoc
4
, 0
3
x y z t = = = = v cc hon v
B.VI. Dn bin bng hm li
Hm li l mt cng c c bn dn bin cc bt ng thc dng c in.
bn c tin theo di chng ri xin nhc li cc kin thc c bn sau y:
1. nh ngha: Mt hm s : , f a b
l

l
l
c gi l li nu:
( ) ( ) ( ) ( ) ( )
1 1 , , , , 0,1 f tx ty tf x t f y x y a b t
l l
_ \
l l
l l

2. Tnh cht:
2.1 Gi s f l hm c o hm n cp 2 trn khong (a,b). Khi f li trn [a,b] khi v ch
khi
( )
''( ) 0, , f x x a b _ \ .
MT S PHNG PHP CHNG MINH BT NG THC
PHNG PHP DN BIN
Trang 39

2.2 Nu hm f li trn [a,b] th f lin tc trn [a,b]. Ngc li , nu f lin tc trn [a,b] th f
li trn [a,b]
( ) ( )
( )
, , ,
2 2
f x f y
x y
f x y a b
1


= _ \


( )

2.3 Bt ng thc Jensen: Gi s f l hm s li trn [a,b]. Khi ta c
(i)
( ) ( ) ( )
1 2
1 2
1 2
...
...
, , ,..., ,
n
n
n
f x f x f x
x x x
f x x x a b
n n
1



l

_ \
l
l


( )

(ii) Vi x
i
l n s thuc on [a,b] v
i
l n s khng m c tng bng 1 ta c:
( ) ( ) ( ) ( )
1 1 2 2 1 1 2 2
... ...
n n n n
f x x x f x f x f x _
Cc bn c th thy tnh cht 2.2 v m rng ra l bt ng thc Jensen c ng dng trc tip
vo phng php dn bin c bit l cc bt ng thc c in. Mt s cc bi ton kh vi
cc dng ton ny l s kt hp gia phng php hm li v cc phng php khc trn cc
min xc nh khc nhau.
Chng ta s cng s dng hm li chng minh mt bt ng thc c in, l bt ng
thc Cauchy

V d B.VI.1: Cho n s thc dng x
i
. Chng minh rng:

1 2
1 2
...
...
n
n
n
x x x
x x x
n
+ + +
>
Chng minh
Lgarit 2 v. Bt ng thc cn chng minh tng ng vi
( ) ( ) ( )
1 2 1 2
ln ln ... ln ...
ln
n n
x x x x x x
n n
+ + + + + + | |
>
|
\ .

Hm s ( ) ln( ) f x x = i t
+
kh vi 2 ln v ( )
2
'' 0, 0 f x x x

= < > . Do vy hm
( ) ( ) g x f x = s tha ( ) '' 0, 0 g x x > > . Vy g l hm li. T p dng BT Jensen ta c
ngay iu phi chng minh.
Bng cch chng minh tng t v p dng nh l 2.3 (ii) ta cng d dng chng minh c
bt ng thc Cauchy dng suy rng.
V d B.VI.2: (V ch Ba Lan 1992) Cho x,y,z tha mn x + y + z = 1. Chng minh rng
2 2 2
9
1 1 1 10
x y z
x y z
+ + s
+ + +
(1)
Chng minh
Xt ( )
2
1
t
f t
t
=
+
, khi ( ) ( ) ( ) (1) 3
3
x y x
f x f y f z f
+ + | |
+ + s
|
\ .

Ta s chng minh f lm hay f li. Ta c ( )
( )
( )
2
3
2
2 3
''
1
t t
f t
t

=
+
nn ''( ) 0, 0, 3 f t t
(
> e

.
Vy nu , , 0, 3 x y z
(
e

th bi ton c gii quyt.
MT S PHNG PHP CHNG MINH BT NG THC
PHNG PHP DN BIN
Trang 40

Trong trng hp cn li th chc chn ta s c du bt ng thc thc s. Do vy c vic
chia thnh nhiu trng hp con xt. n gin, ta gi s x y z > > .
Do 1 x y z + + = v x, y, z khng ng thi | | 0,1 e nn z < 0, suy ra ( ) 0 f z < .
Nu
1
2
y < th
2 2 2
1 2 9
0
1 1 1 2 5 10
x y z
x y z
+ + < + + =
+ + +
nn ta ch xt khi
1
2
y >
Nu
1
0
2
z > > th cng vi
1
2
y > ta c
2 2 2 2 2
4 4 9
1 1 1 5 5 10
1 1
1 1
2 2
x y z x y
z
x y z
+ + s + + = <
+ + +
| | | |
+ +
| |
\ . \ .
, do , ta ch cn xt khi
1
2
z <
Nu
1
3
2
z > > th:
2 2 2
1 1 3 7 9
1 1 1 2 2 10 10 10
x y z
x y z
+ + s + = <
+ + +

Nu 3 z < th: 2 1 4 x x y z > + = > nn x 2 v do
2 2 2
2 1 9
0
1 1 1 5 2 10
x y z
x y z
+ + < + + =
+ + +

Vy ta c iu phi chng minh.
Tt nhin li gii trn cha phi l ngn gn so vi nhiu li gii khc cho bi ton ny
m chng ti c bit. Tuy nhin t tng ca n hon ton trong sng. y, nu thay
v mong mun dn bin ton cc (dn 1 ln 3 bin) bng vic hi vng hp l hn l dn
c 2 bin v bng nhau th li gii s ngn hn. Tht vy, nu c 2 trong 3 bin x, y, z
thuc on 0, 3
(

th dng hm li ta dn c 2 bin ny v bng nhau, v bi ton
ch cn 1 bin, xem nh gii quyt xong. Trong phn cn li th vic chia trng hp s
n gin hn. Nh vy, chng ta c thm mt k thut dn 2 bin v bng nhau l s
dng hm li.
Mc d y l mt cng c tt, nhng mt im yu rt d nhn ra l trong BT, cc
bin phi nm trong cc biu thc c lp nhau ( c th vit thnh dng f(x1) + ...+
f(xn)). Trong khi , nhng BT m ta gp phn ln khng c iu , v ta s phi
lm vic vi dng tng qut hn l f(x1, ..., xn). Chng ta s phi thit lp cc kt qu v
dn bin cho dng tng qut ny mc sau.
K thut dn bin bng hm li cng c th c p dng trong vic dn bin ra bin
nh l: Cho | | : , f a b l mt hm li, Khi ( ) ( ) ( ) { } | | max , , , f x f a f b x a b s e
CHNG MINH
V f lin tc nn f t gi tr ln nht ti | |
0
, x a b e
Xt khi | |
0 0 1 0
| | | | 2 , x a x b x x a a b s = e .
Theo nh ngha hm li ta c: ( ) ( ) ( )
1
1 0
2 2
2
a x
f a f x f f x
+ | |
+ > =
|
\ .
m
( ) ( ) ( ) { } ( ) ( )
0 1 0
max , f x f a f x f a f x > = .
Vi
0 0
| | | | x a x b > chng minh tng t.
Sau y l mt v d p dng
MT S PHNG PHP CHNG MINH BT NG THC
PHNG PHP DN BIN
Trang 41

V d B.VI.3: (USA TST 2004) Cho a b c 0. Chng minh rng
( )
2
3
2 3 a c a b c abc > + +
Chng minh
t ( )
( )
2
3
3 2 f b a b c abc a c = + +
( ) ( )
3
3
2 2
2.
' 1 '' 0
3
ac abc
f b f b
b b
= = > nn f l hm li. p dng nh l trn ta c
( ) ( ) ( ) { }
max , f b f a f c < . Mt khc
( )
( ) ( )
2
3 3 3 3 2 2 2 2
2 3 2 4 0 f a a c a c a c c a c a c a c ac = + = + + + s (BT
Cauchy)
( )
( ) ( )
2
3 3 3 3 2 2 2 2
2 3 2 4 0 f c a c ac a c a ac ac ac ac = + = + + + s (BT
Cauchy)
Vy bi ton c chng minh. Du bng xy ra khi a = b = c
Tng qut ha bi ton trn, ta c mt kt qu kh th v m cch chng minh hon ton
tng t:
Cho
1 2
... 0
n
a a a > > > > . Chng minh rng:
( )
( )
2
1 1 2 1 2
1 ... ...
n
n n n
n a a a a a n a a a > + + +
V d B.VI.4: Cho | | , , , , 0 a b c p q p q e < s . Tm gi tr ln nht ca
a b c
S
b c a c a b
= + +
+ + +

Chng minh
( ) ( )
( ) ( )
2 2
1
'
x y z y z
f x f x
z y x z x y y z
x z x y
= + + =
+ + + +
+ +

( ) ( )
3 3
2 2
''( ) 0
y z
f x
x z x y
= + >
+ +
| | , , , , 0 x y z p q p q e < s
Vy f l hm li. T nh l trn ta d thy S t cc tr khi b s (a, b, c) c k s bng p
v 3-k s bng q.
Theo BT Nesbit th S 3/2. M vi k = 0 hay 3 th S = 3/2, v vy S s t cc tr khi k = 1
hoc k = 2.
t S
1
= S khi k = 1 v S
2
= S khi k = 2. Ta c
( )
1 2
2 3 2 2 3 2
3
2 2
2 2
4 4
0
S S
q p p q
p q q p q p
q p p p q p q q q p
q p
s
+ s +
+ +
+ + s + +
>

Lun ng do q p
Vy gi tr ln nht ca S l
2
2
p q
p q p
+
+
khi a, b, c c 2 s = p v 1 s = q.
MT S PHNG PHP CHNG MINH BT NG THC
PHNG PHP DN BIN
Trang 42

B.VII. Dn bin khng xc nh - UMV
nh l UMV:
Cho ( ) { }
1 2
, ,..., / 0, 1, 2,.., ,
n
n i
D x x x x x i n D c = e > = ng v b chn. Gi Al t hp
cc phn t trong Dc t thnh phn bng 0 v n t thnh phn bng nhau( ) 0 t > .
- Xt 2 php bin i
1 2
, : T T D D nh sau: Vi mi phn t
( )
1 2
, ,..., \
n
a a a a D = e A, chn ra hai ch s sao cho { } min 0, 1, 2,..,
i t
a a t n = > = v
{ }
1 2
ax , ,...,
j n
a m a a a = ,sau thay ,
i j
a a bi
( )
, ,
i j
a a e ( ng vi
1
T ) v
i j
a a ' ' < < < ( ng vi
2
T ).
- : f D lin tc tho mn: ( ) ( ) ( ) ( ) ( ) { }
1 2
min , , f a f T a f T a a D > e
Khi ta c ( ) ( ) { }
min ,
y
f x f y x D
eA
> e
V d B.VII.1: Cho , , 0 a b c > .Chng minh rng:
( ) ( )
3 3 3
9 4 8 , , a b c abc a b c ab bc ca + + + + + + > (1)
Chng minh
t ( ) ( ) ( )
3 3 3
, , 9 4 8 , , f a b c a b c abc a b c ab bc ca = + + + + + +
u tin, chng ta th dn hai bin bng nhau, xt hiu:
( ) ( )
( )
2
, , , , 3 3 9 8
2 2 4
b c
b c b c
f a b c f a b c a

+ + | |
= + +
|
\ .

R rng chng ta cha th c kt lun g. Chng ta tip tc th dn mt bin v 0:
( ) ( ) ( ) , , , , 0 3 3 9 8 f a b c f a b c b c a bc + = + +
R rng, biu thc ( ) 3 3 9 8 b c a + + hoc m, hoc khng m, hay l ta c:
( ) ( ) , , min , , ; , , 0
2 2
b c b c
f a b c f a f a b c
+ + | |
> +
`
|
\ . )

p dng nh l U.M.V: ( ) ( ) ( ) ( ) { }
, , min , , ; , , 0 ; , 0, 0 f a b c f x x x f y y f z >
vi ; ;
3 2
a b c a b c
x y z a b c
+ + + +
= = = + +
Mt khc: ( ) ( ) ( )
2
3 3 2
, 0, 0 4 0; , , 12 24 12 12 1 0 f z z z f x x x x x x x x = + > = + = >
( ) ( ) ( )
2
3 2
, , 0 2 8 8 2 2 0 , , 0. f y y y y y y y f a b c = + = > >
Vy bt ng thc c chng minh, du = xy ra ( ) ( ) ( ) ( ) , , 0, 0, 0 ; 1,1,1 ; 2, 2, 0 a b c =
Cc bn c th nhn thy r rng ngha ca UMV qua bi ton trn, khi m vic hai
kiu dn bin thg thng l dn hai bin bng nhau v y bin ra bin khng t hiu
qu. Nhng s lin h gia 2 k thut ny gip chng ta s dng UMV mt cch ht
sc hiu qu.
MT S PHNG PHP CHNG MINH BT NG THC
PHNG PHP DN BIN
Trang 43

V d B.VII.2: Cho , , , 0 a b c d > tho mn 4 a b c d + + + = . Chng minh rng:
( )
2 4 a b c d abc bcd cda dab + + + > + + + +
Chng minh
t ( )
( )
, , , 2 4 f a b c d a b c d abc bcd cda dab = + + +
Xt cc hiu:
( ) ( )
( )
( )
( )
2
, , , , , , 2 2
2 2 4
a b
a b a b
f a b c d f c d a b a b c d ab
| |
+
+ + | |
| = + + +
|
|
\ .
\ .

( )
( )
( )
( )
( )
( )
2 2
2
8
4 4
2
a b a b
c d c d X
a b a b a b
(

(
+ = +
(
+ + + +
(


( ) ( ) , , , , 0, , f a b c d f a b c d +
( )
( )
( )
( )
4
2 a b a b c d ab c d ab Y c d
ab a b a b
(
(
= + + + = =
(
+ + +


D dng chng minh:
( )
( )
( )
( )
2
4 8
2
ab a b a b
a b a b a b
>
+ + +
+ + + +

Hay Y X > nn ( ) ( ) , , min , , , ; , 0, ,
2 2
c d Y
a b a b
f a b c f c d f a b c d
c d X
+ s + + | |
> +
`
|
+ >
\ . )

p dng nh l UMV ta c:
( ) ( ) ( ) ( )
4 4 4
, , , min 1,1,1,1 ; , , , 0 ; 2, 2, 0, 0 ; 4, 0, 0, 0 0
3 3 3
f a b c d f f f f

| |
> =
`
|
\ . )

ng thc xy ra ( ) ( ) ( ) , , , 1,1,1,1 ; 4, 0, 0, 0 a b c d = .
V d B.VII.3: Cho
1 2
, ,..., 0
n
a a a > tho mn
1 2
... .
n
a a a n + + + = Tm
2 2 2
1 2 1 2
1 2
1 1 1
min ... ... ...
n n
n
S a a a a a a
a a a
| |
= + + + + + + +
|
\ .

Chng minh
t ( )
2 2 2
1 2 1 2 1 2
1 2
1 1 1
, ,..., ... ... ...
n n n
n
f a a a a a a a a a
a a a
| |
= + + + + + + +
|
\ .

Xt cc hiu:
( ) ( )
1 2 1 2 3 1 2 3 4
3 4
1 1 1
, ,..., 0, , ,..., 2 ... ...
n n n
n
f a a a f a a a a a a a a a
a a a
(
| |
+ = + + +
( |
(
\ .
(*)
( )
1 2 1 2
1 2 3
, ,..., , , ,...,
2 2
n n
a a a a
f a a a f a a
+ + | |

|
\ .

MT S PHNG PHP CHNG MINH BT NG THC
PHNG PHP DN BIN
Trang 44

( )
2
1 2
3 4
3 4
1 1 1
2 ... ...
4
n
n
a a
a a a
a a a
( | |
= + + +
( |
(
\ .
(**)

( ) ( )
1 2 1 2
1 2 1 2 3 3
, ,..., min 0, , ,..., , , , ,...,
2 2
n n n
a a a a
f a a a f a a a a f a a
+ + | |
> +
`
|
\ . )

Do vy, p dng nh l UMV ta c: ( )
1 2
1,
, ,..., min , ,..., , 0, 0,..., 0
n
k n
n n n
f a a a f
k k k
=
| |
>
|
\ .

Hay gi tr nh nht ca S l:
1
2 2
min 2 , ,
1 1 2
n
n n n
n
n n n


| |
+
`
|

\ .

)

B.VIII. Dn bin ton min (EMV)
tt c cc php dn bin trc, chng ta thy rng cc bin u c mt i lng
khng i, thng l tng, tng bnh phng, tng hon v hay tch Phng php dn bin
ton min cng c mt i lng khng i, nhng i lng li hon ton mi, l a
b , b c , c a
T tng ch o ca phng php dn bin ton min l chng minh khi tng hoc
gim cc bin i cng mt i lng, bt ng thc yu i hoc khng i. T ch cn
chng minh bt ng thc vi 1 bin bin. iu ny c bit hu dng vi cc bt ng
thc hon v.
Khi dn bin ton min trong bt ng thc n bin, vi 1 bin bin, ta cn phi chng
minh bt ng thc vi n 1 bin kh kh khn. V vy ng dng ch yu ca dn bin ton
min l trong bt ng thc 3 bin.
B.VIII.1 . EMV vi bin ti 0:
V d B.VIII.1: Chng minh vi mi s thc khng m a, b, c ta lun c
( )( ) ( )
3 3 3
3 4 a b c abc a b b c c a + + >
Chng minh
Bt ng thc tng ng vi
( ) ( ) ( ) ( ) ( )( ) ( )
2 2 2
8 a b c a b b c c a a b b c c a
(
+ + + + >

(1)
Khng mt tng qut gi s c = min {a, b, c}. Thay a = a c, b = b c, c = 0 th a b, b c,
c a khng i cn a + b + c gim i. Vy v tri ca (1) gim i cn v phi khng i. Do
ta ch cn chng minh bi ton trong trng hp a, b c = 0. Khi bt ng thc tng
ng vi
( )
3 3
4 a b ab b a + >
iu ny hin nhin ng v
( ) ( )
2 2 2 3 3 3
4 4 4 4 ab a b a b a b ab b a b b a s + s s s +
ng thc xy ra khi a = b = c.
Bi ton trn n gin nhng th hin rt r t tng ca php dn bin ton min. Mt cch
hon ton tng t ta cng chng minh c bi ton vi 4 bin
Chng minh vi mi a, b, c, d khng m
MT S PHNG PHP CHNG MINH BT NG THC
PHNG PHP DN BIN
Trang 45

( )( )( ) ( )
4 4 4 4
4 2 a b c d abcd a b b c c d d a + + + > a
V d B.VIII.2: Cho a, b, c 0 tha mn a + b + c = 3. Chng minh:
2 2 2
4 a b b c c a abc + + + s
Chng minh
Trc tin, mun s dng k thut dn bin ton min, ta phi loi b iu kin bng cch
thun nht ha bt ng thc hay a bi ton tr thnh:
Cho a, b, c 0. Chng minh rng ( )
3
2 2 2
4
27
a b b c c a abc a b c + + + s + +
iu ny
( ) ( )
( )( ) ( )( ) ( )
3
2
8 6 3 27 0
( , , ) 4 4 27 0
a abc b b c bc c b
f a b c a b c b c a b b c c a
+ + >
= + + + >


R rng ta c: { } ( , , ) ( , , ), 0, min , , f a b c f a x b x c s x a b c > e (


V vy ta ch cn chng minh bt ng thc khi c = 0: ( )
3
2
4
27
a b a b s +
p dng bt Cauchy ta c: ( )
3
3
2
4
2 2
4 . . 4
2 2 3 27
a a
b
a a
a B b a b
| |
+ +
|
= s = +
|
|
\ .

Bt ng thc c chng minh
Du = xy ra khi a = b = c = 1 hoc a = 2, b = 1, c = 0 v cc hon v.
V d B.VIII.3: Cho a, b, c 0 i mt phn bit. Chng minh rng
2 2 2
a b b c c a a b b c c a
b c c a a b b c c a a b
+ + + | | | | | |
+ + > + +
| | |
+ + +
\ . \ . \ .
(1)
Chng minh
Gi s c = min{a, b, c} v t
( )
2 2 2
, ,
a b b c c a a b b c c a
f a b c
b c c a a b b c c a a b
+ + + | | | | | |
= + +
| | |
+ + +
\ . \ . \ .

Ta c:
( ) ( )
( ) ( )
( )
( ) ( )
2
c a c b a b
a b b c c a
b c c a a b b c a b c a c b

+ + +
+ + = +
+ + + + + + +

Vy khi gim mi bin i | | 0, x c e do a b, b c, c a khng i nn ( ) , , f a b c gim.
Vy ta ch cn chng minh bt ng thc vi c = 0, tc l
2 2 2
a b b a a b b a
b a a b b a a b
+ | | | | | |
+ + > + +
| | |
+
\ . \ . \ .

Chun ha b = 1, ta cn chng minh: ( )
2
2
2
1 1
1 1
1 1
a a
a a
a a a a
| |
+ + > + + +
|
+
\ .

Kho st hm s trn ta s c iu phi chng minh
B.VIII.2 . EMV vi bin trong tam gic
MT S PHNG PHP CHNG MINH BT NG THC
PHNG PHP DN BIN
Trang 46

V d B.VIII.4: Cho a, b, c l di cnh ca mt tam gic (c th suy bin). Tm gi
tr ln nht ca biu thc: ( ) , ,
a b c
f a b c
b c c a a b
= + +
+ + +

Chng minh
Gi s a = max{a,b,c}. Ta c bin i n gin sau:
( )
( )
( )( )
2
3
, ,
2 2
b c
f a b c
a b a c

= +
+ +


D thy ( ) ( ) | | , , , , , 0, f a b c f a x b x c x x k s e . Trong k l i lng thch hp m
chng ta s tm. T ch cn chng minh bt ng thc trong trng hp mt bin bin.
Th cho x = k = min {a,b,c} tc l cho 1 bin = 0, chng hn c = 0.
Khi ( ) , , 0
a b
f a b
b a
= + . Khng th kt lun g v max trong trng hp ny. l do khi
cho c = 0, ta cng cho lun a = b (do a, b, c l 3 cnh tam gic). V vy ( ) , , f a b c yu i rt
nhiu.
Ch rng, do a = max {a, b, c} nn iu kin cn v a, b, c l 3 cnh tam gic (c th
suy bin) l a b+c. do vy ta nhn thy ngay b + c chnh l bin ca bin a. t ta s y a
v b + c.
Cho x = b + c a 0. Khi
( )
( )( )
3
, , 1 2 2
2 2 2 2
b c bc
f b c b c
c b b c b c c b
+ = + + = s
+ + + +

ng thc xy ra khi a = b, c = 0 v cc hon v. Vy max f(a, b, c) = 2.
V d B.VIII.5: Cho a, b, c l di 3 cnh tam gic. Chng minh rng
4 3 3
a b c a c b
b c a c b a
| | | |
+ + > + + +
| |
\ . \ .
(1)
Chng minh
( )( )( ) ( ) ( ) ( )
2 2 2
(1) 7 6 0
7 0
a b c a c b a c b a b c
b c a c b a c b a b c a
a b b c c a a b c b c a c a b
| | | |
+ + + + + + + + >
| |
\ . \ .
+ + + >

Nu cng gim a, b, c i x 0 th v tri gim.V vy nh bi trn, ta ch cn chng minh bt
ng thc vi c = a + b, tc l
( ) ( )( )
2
3 3 3 3 2 2
2
2 3
7 0 2 2 7 7 0
7 7
2 2 0.
4 8
ab b a a b a b a b a b ab a b
b b a a b a
+ + + + > + + >
| |
+ + >
|
\ .

Bt ng thc c chng minh. ng thc xy ra khi ch khi a = b = c
Qua cc v d trn, c th thy rng ci kh ca EMV chnh l bin i bt ng thc v
dng cha cc bin a b, b c, c a ph hp. Vic ny i hi mt qu trnh rn luyn
lu di. Tuy nhin mt khi bin i xong. Vic p dng EMV tr nn kh d dng.
MT S PHNG PHP CHNG MINH BT NG THC
PHNG PHP DN BIN
Trang 47

B.IX. nh l dn bin tng qut {GMV
GENERAL MIXING VARIABLES}
Trong mc ny, chng ti s gii thiu nh l GMV dng dn bin cho n s. y l mt
nh l rt tng qut, bao gm gn nh ton b cc kh nng ca dn bin.
1. Chng ta bt u bng mt s nh ngha trong khng gian
n
.
nh ngha 1:
Khng gian
n
l tp hp cc b th t ( )
1 2
, ,...,
n
x x x x = vi , .
n
i
x i e
Mt dy
( ) { } 1, 2, ,
, ,...,
m m m n m
x x x x = trong
n
gi l hi t v ( )
1 2
, ,...,
n
n
z z z z = e nu
tng dy
, i m
x hi t v
i
z khi , 1, 2,..., m i n = .
Cho .
n
D c Mt hm s : f D gi l lin tc trn D nu : Vi mi d
{ }
m
x D c v vi mi z D e sao cho { }
m
x hi t v z th ta u c: ( )
m
f x hi t v
( ) f z
nh ngha 2: Cho
n
D c . Ta ni:
D ng nu vi mi dy v vi mi
n
z e sao cho { }
m
x hi t v z th ta u c
z D e .
D b chn nu tn ti s thc M sao cho: ( )
1 2
, ,...,
n
x x x x D = e
th , 1, 2,...,
i
x M i n s = .
nh l Welerstrass: Cho D ng v b chn trong
n
v :
n
f D lin tc. Th f t
gi tr nh nht trn D, ngha l tn ti
0
x D e sao cho: ( ) ( )
0
, . f x f x x s e Chng ta
cng c nhng kt qu tng t vi gi tr ln nht.
Bnh lun: nh l ny l mt m rng ca mt kt qu kh quen thuc: Cho | | , a b l 1
khong ng trong v | | : , f a b lin tc th f c gi tr nh nht trn | | , a b .
Do , v mt trc gic th nh l 1 kh r rng.
2. nh l GMV: Ta s lun gi thit rng:
D l mt tp con trong
n
, v A l mt tp con ng ca D.
: f D l mt hm s lin tc sao cho f c gi tr nh nht trn A .

1 2
, ,..., :
k
T T T D D l cc php bin i sao cho ( ) ( )
1
... ,
k
T x T x x x = = = eA
Ta s t ra cc tiu chun gi tr nh nht ca f trn A cng chnh l gi tr nh nht
ca f trn D. Ta c:
nh l GMV1: Nu
( ) ( ) ( ) { }
1,
min , \
j
j k
f x f T x x D
=
> e A
1, , j k x D = e ta c ( ) lim
m
j
m
T x

eA , trong ( ) ( ) ( ) ( )
0 1
,
m m
j j j j
T x x T x T T x

= =
Th ( ) ( ) { }
min ,
y
f x f y x D
eA
> e .
nh l GMV2: Nu
MT S PHNG PHP CHNG MINH BT NG THC
PHNG PHP DN BIN
Trang 48

D ng v b chn trong
n
.
( ) ( ) ( )
1,
min , \
j
j k
f x f T x x D
=
> e A
Th ( ) ( ) { }
min ,
y
f x f y x D
eA
> e , hn na ng thc khng xy ra trn \ D A.
nh l GMV3: Nu
D ng v b chn trong
n
.
( ) ( ) ( )
1,
min , \
j
j k
f x f T x x D
=
> e A
Tn ti cc hm s
j
h lin tc tho mn: ( ) ( ) ( )
, \
j j j
h x h T x x D > e A.
Th ( ) ( ) { }
min , .
y
f x f y x D
eA
> e
Chng minh GMV1: Ly tu . Ta c: ( ) ( ) ( )
1,
min
j
j k
f x f T x
=
> v bng quy np suy ra
( ) ( ) ( )
1,
min ,
m
j
j k
f x f T x x D
=
> e . Do f lin tc v ( ) lim
m
m
T x

eA nn:
( ) ( ) ( ) ( )
( )
( )
1,
1,
lim min min lim min
m m
j j
m m y j k
j k
f x f T x f T x f y
eA =
=
| |
> = > |
|
\ .

Hn na, nu ( ) ( ) ( )
1,
min
j
j k
f x f T x
=
> th do ( ) ( ) ( ) min
j
y
f T x f y
eA
> nn ( ) ( ) min
y
f x f y
eA
>
Chng minh GMV2: Do nh l Weierstrass, tn ti
0
x D e sao cho ( ) ( )
0
, f x f x x D s e .
Nu
0
x eA th ( ) ( ) ( )
0 0
1,
min
j
j k
f x f T x
=
> , mu thun. Vy
0
x eA v ta c pcm.
Chng minh GMV3: Ly
0
y eA sao cho ( ) ( ) { }
0
min
y
f y f y
eA
= . Gi s phn chng rng tn
ti z D e sao cho ( ) ( )
0
. f z f y < Do ( )
j
h x b chn di trn D nn bng cch cng thm
mt hng s khi cn, ta c th gi s ( ) 0, , 1, 2,...,
j
h x x D j k > e = .
Chn 0 > nh ta c: ( ) ( ) ( )
0
1
k
j
j
f z h z f y
=
+ <


t ( ) ( ) ( ) { }
1,
min ,
j
j k
g x f x h x x D
=
= + e th : g D lin tc v hn na
( ) ( ) ( )
1,
min , \
j
j k
g x g T x x D
=
> e A v ( ) ( ) ( ) { }
0
min
y
g z f y g y
eA
< s .
iu ny mu thun vi nh l 2, hay gi thit phn chng l sai, suy ra pcm.
Bnh lun: Tuy hnh thc pht biu ngn gn nhng GMV c tm ng dng cc k rng
ri. C mi mt ( hay mt vi ) php bin i T thch hp l ta li c mt tiu chun dn
bin. Chng hn, ta c ngay hai h qu sau y.
H qu 1:[Phm Kim Hng, SMV Strongly Mixing Variables] Cho:

n
D R c l mt tp ng, b chn v ( )
0
, ,..., s s s s D = e .
Php bin i T nh sau: Vi mi b n s ( )
1 2
, ,...,
n
a a a , ta chn ra s ln nht v s
nh nht ri thay th bng trung bnh cng ca chng. Gi s rng : T D D .
: f D l mt hm s lin tc, i xng tho mn: ( ) ( ) ( )
, f a f T a a D > e .
MT S PHNG PHP CHNG MINH BT NG THC
PHNG PHP DN BIN
Trang 49

Khi : ( ) ( )
0
, f a f s a D > e
Chng minh: Chn ( )
2
1 2
1
, ,...,
n
n i
i
h a a a a
=
=

th d thy : h D lin tc v
( ) ( ) ( )
0
, \ h a h T a a D s > e . p dng nh l GMV3 ta c pcm.
Ghi ch:
Ta cng c th p dng GMV1, khi d ch cn kim tra vi mi a D e th ( )
0
lim
m
m
T a s

= .
S kin ny kh r rng v mt trc gic, v ngh bn c t chng minh. Vi cch lm
na, chng ta c th b qua gi thit T ng v b chn (ngha l H qu 1 trn ng vi mi
tp con D ca
n
sao cho ( ) T D D c )
H qu 2: [inh Ngc An, UMV Unefine Mixing Variables] Cho:
( )
1 2
1
, ,..., 0, onst
n
n
n i i
i
D x x x x x x c
=

c = e > =
`
)

. Gi A l t hp c t thnh phn
bng 0 v n t thnh phn bng nhau ( ) 0 t > .
: f D lin tc, i xng tho mn:
( ) ( )
1 2 1 2
1 2 3 1 2 3
, ,..., min , , ,..., , 0, , ,...,
2 2
n n n
a a a a
f a a a f a a f a a a a
+ + | |
> +
`
|
\ . )

Th ( ) ( ) { }
min ,
y
f x f y x D
eA
> e
Chng minh: Chn 2 php bin i
1 2
, : T T D D nh sau: ( ) ( )
2 2
, T a T a a a = = eA v
vi mi phn t ( )
1 2
, ,..., \
n
a a a a D = A, chn ra 2 ch s i sao cho
{ } min 0 1, 2,...,
i t
a a t n = > = avf { }
1 2
ax , ,...,
j n
a m a a a = , ng vi
1
T ta thay ,
i j
a a bi trung
bnh cng ca chng, ng vi
2
T ta thay ,
i j
a a bi
( )
0,
i j
a a + . Th ta c
( ) ( ) ( ) ( ) ( ) { }
1 2
min , , f a f T a f T a a D > e
Chn ( )
2
1 1 2
1
, ,...,
n
n i
i
h a a a a
=
=

v th
1 2
, : h h D lin tc v
( ) ( ) ( ) { }
0
, \ , 1, 2,...
j j j
h a h T a a D s j n > e = . p dng GMV3 ta c pcm.
Ghi ch: Ta cng c th kim tra vi mi a D e th ( ) ( )
2
0
lim 0,..., 0, ,...,
m
m
T a r r

=
ri p dng GMV1.
Do y l mt phn kh kh nn ta s ch xt mt v d sau:
V d B.IX.1: (Bt ng thc Cauchy) Cho n s thc khng m x
1
,x
2
,,x
n
. Chng minh rng
1 2 1 2
... ...
n
n n
x x x n x x x + + + >
Chng minh
Chun ha
1 2
... 1
n
x x x = v chng minh
1 2
...
n
x x x n + + + > .
Tt nhin ta ch xt khi ,
i
x n i s
MT S PHNG PHP CHNG MINH BT NG THC
PHNG PHP DN BIN
Trang 50

Xt ( ) | | { }
1 2 1 2
, ,..., | 0, , ... 1
n i n
D x x x x x n x x x = = e = th d thy D ng v b chn.
Xt ( ) { }
0
1,1,1...,1 x A = = . Xt : f D lin tc nh sau:
Vi mi ( )
1 2
, ,...,
n
x x x x D = e th ( )
1 2
...
n
f x x x x = + + +
Xt : \ T D D A nh sau: Vi mi ( )
1 2
, ,..., \
n
x x x x D = e A th tn ti
i j
x x = v ta t
T(x) l b thu c t x sau khi thay x
i
v x
j
bi trung bnh nhn ca chng. Khi d thy
( ) ( ) ( ) ( )
2
0
i j
f x f T x x x = > . Vy ta c th p dng GMV2 suy ra
( ) ( )
0
, f x f x x D > e , hn na du = ch xy ra khi x = x
0
.
B.X. Bi tp

Bi B.X.1: Cho , , 0 a b c > tho mn 1 abc = . Chng minh rng:
2 2 2
3 1 1 1
6
2
a b c a b c
a b c
| |
+ + + > + + + + +
|
\ .

Chng minh
t ( )
2 2 2
3 1 1 1
, , 6
2
f a b c a b c a b c
a b c
| |
= + + + + + + + +
|
\ .

Khng mt tnh tng qut ta gi s { } min , , a a b c =
Xt hiu ( )
( )
, , , , d f a b c f a bc bc =
( )
( )
( )
( ) ( )
2
2 2 2
2 3 3 3 3
2 2 2 2
b c
b c b c b c b c
bc bc

(
= = +
(

(1)
Do { } min , , a a b c = v 1 abc = nn
1 bc >
( ) ( )
2 2
3 3 3 3 3 3
4, 4 1 0
2 2 2 2 2 2
b c b c
bc bc
+ > s + > = >
Nn t (1) suy ra ( ) ( ) 0 , , , , d f a b c f a t t > > (vi
2
, 1 a t at s = )
Xt ( ) ( )
2 2
3 3 1 2
, , 2 6 2
2 2
f a t t a t a t
a t
| |
= + + + +
|
\ .

( )
2
6 5 4 3 2
2 4 4
3 3 1 1
3 6 6 12 6 3 2
2 2 2
t
t t t t t t
t t t t
= + + = + +
( ) ( )
2
4 3 2
1 4 3 4 2 t t t t t = + + + (2)
t ( ) | )
4 3 2
4 3 4 2, 1, g t t t t t t = + + + e +
C ( ) ( ) ( ) ( )
3 2
3
4 3 1 4 2
2
g t t t t t x t y t
| |
' = + + =
|
\ .
(Vi 0,1 2 x y < < < )
( ) 0
2
t y
g t
t
=
' =

=


Xt bng bin thin
MT S PHNG PHP CHNG MINH BT NG THC
PHNG PHP DN BIN
Trang 51

t 1
y
2 +
( ) g t '
2 + 0 - 0 +
( ) g t


6


( ) g y




5


+


Vy t Bng bin thin ta c ( ) | ) 5 0 / 1, g t > > + nn t (2) ( ) , , 0 f a t t > hay
( ) , , 0 f a b c > (pcm)
Du = xy ra khi 1 a b c = = =
Bi B.X.2: Cho , , 0, 3 a b c a b c > + + = Chng minh rng:
( )( ) ( )
3 3 3 3 3 3 3 3 3
36 a b c a b b c c a ab bc ca + + + + s + +
Chng minh
Khng mt tng qut gi s a b c > > . t
( ) ( ) ( )( )
3 3 3 3 3 3 3 3 3
, , 36 f a b c ab bc ca a b c a b b c c a = + + + + + +
Khi ( ) ( ) ( )
( )
( )
3 3
3 3
, , 0 36 f a b c a b c a b c a b c + = + + + +
M ( ) ( ) 36 36 a b c ab bc ac + s + +
( )
( )
3
3 3 3 3
3
3 3 3 3 3 3 3
a b c a b c
a b b c c a a b c
+ + s + +
+ + s +

( ) ( ) , , , , 0 f a b c f a b c > +
Do ta ch cn chng minh bt ng thc trong trng hp c = 0 hay
( ) ( )
3 3 3 3 3 3 2 2
36 36 ab a b a b a b a b > + > +
t t = ab, a + b = 3 nn bt ng thc cn chng minh tng ng vi
( ) ( )
2 2 3 2
36 27 9 6 3 4 3 t t t t t t > > + > vi
9
0,
4
t
(
e
(

iu ny hin nhin ng theo
bt Cauchy cho 3 s khng m
3 3
, , 4
2 2
t t

Vy bi ton c chng minh, du bng xy ra khi c = 0, ab = 2 , a + b = 3 v cc hon v
1, 2, 0 a b c = = = v cc hon v

MT S PHNG PHP CHNG MINH BT NG THC
BT NG THC SCHUR V PHNG PHP BIN I PQR
Trang 52


C BT NG THC SCHUR
V PHNG PHP BIN I
PQR
C.I. Bt ng thc Schur
C.I.1 . L thuyt
a) Bt ng thc Schur dng chnh tc
Cho a,b,c 0, k l s thc bt k th ta lun c
( )( ) ( )( ) ( )( ) 0
k k k
a a b a c b b c b a c c a c b + + >
CHNG MINH
Khng mt tnh tng qut gi s 0 a b c > > > khi ta c
VT = ( )( ) ( )[ ( ) ( )] 0
k k k
c c a c b a b a a c b b c + >
iu ny hin nhin ng. Bt ng thc Schur c chng minh
Du = xy ra a=b=c hoc a=b,c=0 cng cc hon v.
Cc trng hp thng gp ca Schur
Vi k=0
2 2 2
( )( ) ( )( ) ( )( ) 0
( )( ) ( )( ) ( )( ) 0
k k k
a a b a c b b c b a c c a c b
a b a c b c b a c a c b
a b c ab bc ca
+ + >
+ + >
+ + > + +

Vi k=1
2
3 3 3
( )( ) ( )( ) ( )( ) 0
( )( ) ( )( ) ( )( ) 0
3 ( ) ( ) ( )
k k
a a b a c b b c b a c c a c b
a a b a c b b c b a c c a c b
a b c abc ab a b bc b c ca c a
+ + >
+ + >
+ + + > + + + + +

Vi k=2
2 2 2
4 4 4 2 2 2 2 2 2
( )( ) ( )( ) ( )( ) 0
( )( ) ( )( ) ( )( ) 0
( ) ( ) ( ) ( )
k k k
a a b a c b b c b a c c a c b
a a b a c b b c b a c c a c b
a b c abc a b c ab a b bc b c ca c a
+ + >
+ + >
+ + + + + > + + + + +

b) Cc dng suy rng ca bt ng thc Schur
nh l 1: Bt ng thc Schur suy rng 1(Vornicu Schur)
Phn
C
MT S PHNG PHP CHNG MINH BT NG THC
BT NG THC SCHUR V PHNG PHP BIN I PQR
Trang 53

Xt bt ng thc ( )( ) ( )( ) ( )( ) 0 x a b a c y b c b a z c a c b + + > (1)
Khi (1) ng vi mi 0 a b c > > > v , , 0 x y z > nu tho mn 1 trong cc kh nng sau:
1. x y > hoc y z >
2. ax by >
3. bz cy > (nu a,b,c l 3 cnh ca 1 tam gic)
4. x y z + >
5.
2 2 2
2( ) x y z xy yz xz + + s + +
CHNG MINH
Nu x y > th ta c ( )( ) ( )[ ( ) ( )] 0 VT z a b b c a b x a c y b c = + >
Nu z y > th ta c ( )( ) ( )[ ( ) ( )] 0 VT x a b a c b c z a c y a b = + >
Vi 0 a b c > > > th ( )
a
a c b c
b
> V vy ta c
( )( )( )
( )( ) ( )[ ( ) ( )] ( )( ) 0
a b b c ax by
VT z a b b c a b x a c y b c z a b b c
b

= + > + >
Do 0 a b c > > > v a,b,c l di 3 cnh ca 1 tam gic nn ( )
b
a c a b
c
>
( )( ) ( )[ ( ) ( )]
( )( )( )
( )( ) 0
VT x a b a c b c z a c y a b
b c a b bz cy
x a b a c
c
= +

> + >

4. Ta c

2
( )( ) ( )( ) ( )( ) 0
( )( ) ( )( ) ( )( )
. .
( . . )
2
( )
x a b a c y b c b a z c a c b
x a b a c z c a c b y b c a b
a c a c
x z y
b c a b
a b b c
x z x z y
b c a b
x z xz y
x z y
x z y
+ + >
+ >

+ >


+ + + >

+ + >
+ >
+ >

Ta c
2 2 2
2 2 2
2 2
( )( ) ( )( ) ( )( ) 0
( )( ) ( )( ) ( )( ) 0
( )( ) ( )[( ) ( )] ( )( ) 0
( ) ( )( )( ) ( ) 0(*)
x a b a c y b c b a z c a c b
x y z a b y z x b c z x y c a
x y z a b y z x a b c a z x y c a
y a b y z x c a a b z c a
+ + >
+ + + + + >
+ + + + + + >
+ + + >

Nu y=0 th

2 2 2 2 2
2
2( ) 2
( ) 0
x y z xy yz xz x z zx
z x z x
+ + s + + + s
s =

Khi
2
(*) ( ) 0 z c a > (lun ng)
MT S PHNG PHP CHNG MINH BT NG THC
BT NG THC SCHUR V PHNG PHP BIN I PQR
Trang 54

Nu 0 y = ta coi VT l tam thc bc hai vi bin (a - b). chng minh (*)ng ta chng
minh 0 A s . Tht vy

2 2 2
2 2 2 2
0 ( ) ( ) 4 ( ) 0
( ) 4 0 2( )
y z x c a yz c a
y z x yz x y z xy yz xz
A s + s
+ s + + s + +

(Do cc nh l t nh l 2 tr i khng c dng nhiu trong chuyn nn ch c vit
mang tnh gii thiu m khng c chng minh c th.)
nh l 2: Cho p,q,x,y,z,a,b,c l cc s dng tho mn (a,b,c) v (x,y,z) l cc b
n iu cng chiu.Khi ta lun c
[( )( ) ( )( )] [( )( ) ( )( )]
[( )( ) ( )( )] 0
p p q q q q p p p p q q q q p p
p p q q q q p p
x a b a c a b a c y b c b a b c b a
z c a c b c a c b
+ + +
+ + >

Cc bt ng thc h qu: Cho , , , , , , 0 a b c m n p q > Ta c
[ ( )( ) ( )( )] 0
m p p q q m q q p p
cyc
a a b a c a a b a c + >


( )
( )( ) 0
m n n m p p p p
cyc
a b a b c a c a + >


( ) ( )( ) ( )( )
0
m n n m p p p p q q q q q q q q p p q q
cyc
a b a b a b a c a b a c a b a c a b a b
(
+ + >


nh l 3: Xt bt ng thc sau, trong mi hm s f : ta c
( ) ( ) ( ) ( ) 0 f a b f a c f a d f a e >


Bt ng thc ng th hm s f phi tho mn 2 iu kin sau :
f l hm n iu tng
. ( ) 0 x f x x > e
nh l 4: Bt ng thc Schur m rng cho tam gic
Cho tam gic ABC v A
1
B
1
C
1
c cc cnh ln lt l a,b,c v a
1
,b
1
,c
1
.Khi ta c:

1 1 1
( )( ) ( )( ) ( )( ) 0 1 a bc a b a c b ac b c b a c ab c a c b

+ + > >
Mnh : Cho M l mt im nm trong tam gic ABC. Cc ng thng AM, BM, CM ct
cc cnh BC, CA, AB ln lt ti A
1
, B
1
, C
1
.Khi (a.MA.MA
1
, b.MB.MB
1
, c.MC.MC
1
) l
di 3 cnh ca 1 tam gic.
S dng mnh trn v nh l 4 ta c cc h qu sau

2
( )( ) 0
( )
a
cyc
a b c a
cyc cyc
m
a b a c
a
a m bc m m m
>
> +



( )( ) 0
cyc
x
x y x z
y z
>
+


MT S PHNG PHP CHNG MINH BT NG THC
BT NG THC SCHUR V PHNG PHP BIN I PQR
Trang 55


( )( )
( ) ( )
( )( )
( ) ( )
1
. 0
0
0
cos 0
cyc
a a
cyc
cyc
cyc
MAMA a b a c
m h a b a c
IA a b a c
A a b a c
>
>
>
>


C.I.2 . V d
V d C.I.1: Bt ng thc Nebit
3
2
a b c
b c a c a b
+ + >
+ + +
(*)
Chng minh

( )( ) ( )( ) ( )( ) ( )( )
( ) ( ) ( )
3 3 3
(*) 2[ ] 3( )
3 (1)
a a b a c b b c b a c c a c b a b b c c a
a b c abc ab a b bc b c ca c a
+ + + + + + + + > + + +
+ + + > + + + + +

V (1) lun ng theo bt ng thc Schur bc 1 nn bt ng thc Nebit c chng minh.
Nhn xt: Vi bt ng thc Nesbit ta c rt nhiu cch chng minh khc nhau nhng
vic quy ng mu s vi cc bt ng thc 3 bin dng n gin l suy ngh rt t nhin
v cch chng minh sau khi quy ng l v cng n gin.Tuy nhin khng phi bt ng
thc no cng c p dng bt Schur trc tip nh vy.Sau y s tm hiu mt bt ng
thc nh vy
V d C.I.2: Cho , , 0 a b c > CMR
3 3 3
2 2 2 2 2 2
3( ) a b c ab bc ca
A
b bc c c ca a a ab b a b c
+ +
= + + >
+ + + + +

Chng minh
p dng bt ng thc Bunhiacopxki ta c
( ) ( ) ( ) ( )
( ) ( ) ( )
( )
2
2 2 2 2 2 2 2 2 2
2 2 2 2
2 2 2 2 2 2
2 2 2 2
.
( )
( )
( ) ( ) 3
A a b bc c b c ca a c a ab b a b c
a b c
A
a b bc c b c ca a c a ab b
a b c
A
ab a b bc b c ca c a abc
(
+ + + + + > + +

+ +
>
+ + + + +
+ +
>
+ + + + +

Mt khc ta d dng nhn thy
2
( ) 3( ) a b c ab bc ca + + > + +
Nn ta cn chng minh

( ) ( )
( ) ( ) ( ) ( ) ( )
( )
( ) ( )
2 2 2 2
2
2 2 2
2
4 4 4 2 2 2 2 2 2 2 2
4 4 4 2 2 2 2 2 2
( )
( )
3
2( ) ( ) ( ) ( )
( ) ( ) (*)
a b c
a b c
ab a b bc b c ca c a
a b c a b c ab a b bc b c ca c a abc
a b c a b b c c a abc a b c ab a b bc b c ca c a
a b c abc a b c ab a b bc b c ca c a
+ +
> + +
+ + + + +
+ + > + + + + + + + (

+ + + + + > + + + + + + + +
+ + + + + > + + + + +

MT S PHNG PHP CHNG MINH BT NG THC
BT NG THC SCHUR V PHNG PHP BIN I PQR
Trang 56

Bt ng thc (*) lun ng theo bt ng thc Schur bc 2
pcm
Du = xy ra a=b=c
V d C.I.3: Cho , , 0 a b c > v 3 a b c + + = CMR
3
(*)
2
a b c
a bc b ac c ab
+ + >
+ + +

Chng minh
Ta c

( ) ( )
( )
( )
( ) ( ) ( )
( )( )
( )
3 3( ) 2 ( )
2 2 2
2 ( )
(*)
2
2 ( )
0
2
2 2
0
3 2 2
0
(
cyc
cyc cyc
cyc
cyc
cyc
ab bc ca a b c bc
ab bc ca ab bc ca
a a b c bc
a bc ab bc ca
a a b c bc
a bc ab bc ca
a ab bc ca a bc a b c bc
a bc a b c
bc a bc ab ac
a bc
bc a
+ + +
= =
+ + + +
+
>
+ + +
| |
+
>
|
|
+ + +
\ .
+ + + + (

>
+ + +
+
>
+
+

| | ) 2 2
0
( )( )
0
( )( )
0(1)
( )
cyc
cyc
cyc
b c a bc ac ab
a bc
bc a c a b
a bc
a b a c
a a bc
+ +
>
+

>
+

>
+


t
( )
1 1 1
, ,
( ) ( )
x y z
a a bc b b ca c c ab
= = =
+ + +
v , , 0 a b c > nn , , 0 x y z > v ta c
( )( ) ( )( ) ( ) ( ) (1) 0(2) x a b a c y b c b a z c a c b + + >
Mt khc khng mt tnh tng qut ta gi s a b c > > khi ta c

( ) ( )
1 1
c a bc b b ac
>
+ +

Nn (2) ng theo nh l Schur suy rng
pcm
Du = xy ra a b c = =
V d C.I.4: Cho , , 0 a b c > .Chng minh rng
2 2 2
2 2 2 2 2 2
0
a bc b ac c ab
b bc c c ac a a ab b

+ + >
+ + + + + +

Chng minh
Ta c
MT S PHNG PHP CHNG MINH BT NG THC
BT NG THC SCHUR V PHNG PHP BIN I PQR
Trang 57

( )( ) ( )
( )( )
2 2
2 2 2 2 2 2
2
2 2 2 2 2 2
2
cyc cyc cyc
cyc cyc
a bc a ac ab bc ac ab bc
b bc c b bc c b bc c
a b a c ab a b
b bc c a ac c b bc c
+ +
= +
+ + + + + +

= +
+ + + + + +



V
( )
( )( )
2
2 2 2 2
0 , , 0
cyc
ab a b
a b c
a ac c b bc c

> >
+ + + +

nn ta cn chng minh
( )( )
2 2
0
cyc
a b a c
b bc c

>
+ +

(*)
Mt khc khng mt tnh tng qut ta gi s a b c > > khi ta c

( )( )
( )( )
2 2 2 2 2 2 2 2
1 1
0
a b a b c
b bc c a ac c a ac c b bc c
+ +
= >
+ + + + + + + +
( a b c > > )
Nn theo nh l Schur suy rng ta c(*) lun ngpcm
Du = xy ra a b c = =
V d C.I.5: Cho , , 0 a b c > .Chng minh rng
3 3 3
a abc b abc c abc
a b c
b c a c a b
+ + +
+ + > + +
+ + +

Chng minh
Ta c:

( )
( ) ( ) ( )
( ) ( )
3 3 3
3
2
cyc
a abc b abc c abc
A a b c
b c a c a b
a abc
a
b c
a
a b a c
b c a bc b c a
+ + +
= + + + +
+ + +
| |
+
= |
|
+
\ .
=
+ + + +


t

( ) ( ) ( )
( )( ) ( )
( )( ) ( )
2
2
2
,
,
a
x
b c a bc b c a
b
y
a c b ac a c b
c
z
a b c ab a b c
=
+ + + +
=
+ + + +
=
+ + + +

Ta c ( )( ) ( )( ) ( )( ) A x a b a c y b c b a z c a c b = + +
Mt khc khng mt tnh tng qut ta gi s 0 a b c > > > th ta c
MT S PHNG PHP CHNG MINH BT NG THC
BT NG THC SCHUR V PHNG PHP BIN I PQR
Trang 58

( )( ) ( ) ( )( ) ( )
( )( ) ( )( )
( )( ) ( )( )
( )
( ) ( ) ( )
( )( ) ( )( )
( )
2 2
2 2
2 2
2 2 2
2 2
0
0
0(*)
x y
a b ac c a c a ab b a bc b c b c bc
a b ac c a b a bc b c
a b ab
a b ac c a b a bc b c
a b c a ab b c a b abc
a b ab
a b ac c a b a bc b c
>
+ + + + + + + >
+ + + +
+ >
+ + + + +
(
+ + + +

+ >
+ + + + +

V 0 a b c > > > nn
( ) ( )
2 2
2 2 0 c a ab b abc c ab ab abc abc + + > + = >
( ) * lun ng
Nn theo nh l Schur suy rng ta c pcm
Du = xy ra a b c = = hoc 0 a b = > , 0 c = cng cc hon v
V d C.I.6: Cho , , 0 a b c > CMR
( ) ( ) ( )
3 3 3
3 3 3
3
2
a abc b abc c abc
b c a c a b
+ + +
+ + >
+ + +

Chng minh
V
3
2
a b c
b c a c a b
+ + >
+ + +
nn ta cn chng minh
( ) ( ) ( )
( ) ( )
( )
( )( )
3 3 3
3 3 3
2
0
cyc
a abc b abc c abc a b c
b c a c a b
b c a c a b
a
a b a c
b c b c a bc a b c
+ + +
+ + > + +
+ + +
+ + +
>
+ + + + +


Khng mt tnh tng qut gi s a b c > > khi ta c
( ) ( ) ( )( )
( ) ( )
( ) ( ) ( ) ( )
2 2
2
0
. . (1)
b c a a b c a b ab c
c a b b c a
c a c a b c a b c b c a b c
+ + = >
+ > +
+ + + > + + +

V ta c
( ) ( ) ( ) ( ) ( )( )
( ) ( )
( ) ( )
2 2
2 2 2 2 2
2 2
2 2
0
. . (2)
c a b ac b c a bc c a b a b c ac bc ab
c a b ac b c a bc
c a c a b ac b c b c a bc
+ + + + = + + + + >
+ + > + +
+ + + > + + +

T (1) v (2)
( ) ( )
( )
( ) ( )
( )
( ) ( )
( )
( ) ( )
( )
2 2
2 2
b c b c a bc a b c c a c a b ac b a c
a b
b c b c a bc a b c c a c a b ac b a c
+ + + + + s + + + + +
>
+ + + + + + + + + +

theo nh l Schur suy rng bt ng thc (*)c chng minh
MT S PHNG PHP CHNG MINH BT NG THC
BT NG THC SCHUR V PHNG PHP BIN I PQR
Trang 59

Du = xy ra a b c = =
V d C.I.7: Cho , , a b c eChng minh rng
( )
6 6 6 2 2 2 3 3 3 3 3 3
3 2 a b c a b c a b b c c a + + + > + +
Chng minh
V
6 6 6 2
6 6 6 2 2 2
3 3 a b c a b c a b c abc + + + = + + +
V
( ) ( )
3 3 3
3 3 3 3 3 3
2 2 a b b c c a ab bc ca + + s + +
Nn ta ch xt trng hp , , 0 a b c >
Theo bt ng thc Schur ta c
( )( ) ( )( ) ( )( )
( ) ( ) ( )
2 2 2 2 2 2 2 2 2 2 2 2 2 2 2
6 6 6 2 2 2 2 2 2 2 2 2 2 2 2 2 2 2
0
3
a a b a c b b c b a c c a c b
a b c a b c a b a b b c b c c a c a
+ + >
+ + + > + + + + +

Nn ta cn chng minh
( ) ( ) ( ) ( )
( ) ( ) ( )
2 2 2 2 2 2 2 2 2 2 2 2 3 3 3 3 3 3
2 2 2
2 2 2 2 2 2
2
0
a b a b b c b c c a c a a b b c c a
a b a b b c b c c a c a
+ + + + + > + +
+ + >

Bt ng thc trn lun ng nn ta c pcm
Du = xy ra a b c = = hoc , 0 a b c = = cng cc hon v

C.II. Phng php bin i pqr
C.II.1 . a thc i xng 3 bin
a) Khi nim:
Gi s F(a,b,c) l a thc vi b 3 bin a,b,c
Ta gi F(a,b,c) l a thc i xng 3 bin nu F(a,b,c)=F(a,b,c) trong (a,b,c) l hon
v tu ca (a,b,c).
b) a thc i xng Viete
nh ngha: , , p a b c q ab bc ca r abc = + + = + + = c gi l a thc i xng
Viete
Mnh : Mi a thc i xng F(a,b,c) u biu din di dng T(p,q,r).
Cc hng ng thc ng nh
MT S PHNG PHP CHNG MINH BT NG THC
BT NG THC SCHUR V PHNG PHP BIN I PQR
Trang 60


( ) ( )( )
( ) ( ) ( )( ) ( )( )
( ) ( ) ( )
( ) ( ) ( )
2
2 2 2 2
3 3 3 3
2 2 2 2 2 2 2
3 3 3 3 3 3 3 2
4 4 4 4 2 2
2 2 2 2 2 2 2 2
2
3 3
2
3 3
3
4 2 4
2
a b b c c a pq r
a b a c b c b a c a c b p q
a b c p q
a b c p pq r
a b b c c a q pr
a b b c c a q pqr r
ab a b bc b c ca c a pq r
a b c p p q q pr
ab a b bc b c ca c a p q q pr
a
+ + + = +
+ + + + + + + + = +
+ + =
+ + = +
+ + =
+ + = +
+ + + + + =
+ + = + +
+ + + + + =
4 4 4 4 4 4 4 2 2 2 2
4 2 4 b b c c a q pq r p r qr + + = + +

C.II.2 . Xy dng cc bt ng thc ca pqr
a) Xy dng cc bt ng thc t bt ng thc Schur
Vi k=0

2 2 2
2
3
a b c ab bc ca
p q
+ + > + +
>

Vi k=1

( ) ( ) ( )
3 3 3
3
3
3
3 3 3 3 0
4 9 0
a b c abc ab a b bc b c ca c a
p pq r r pq r
p pq r
+ + + > + + + + +
+ + + >
+ >

Vi k=2

( ) ( ) ( ) ( )
4 4 4 2 2 2 2 2 2
4 2 2 2 2
4 2 2
4 2 4 2 0
5 4 6 0
a b c abc a b c ab a b bc b c ca c a
p p q q pr pr p q q pr
p p q q pr
+ + + + + > + + + + +
+ + + + + >
+ + >

b) Xy dng t cc bt ng thc khc

2 2 2 2 2 3 2
2 2 2 2 2 3 2
3 4 2 2 3 3 3
1, 3 4, 3 4 7, 3 2 , 10, 9 4
2, 3 5, 3 4 8, 2 3 ,11, 2 9 7
3, 27 6, 3 4 9, 2 9 7 , 12, 6
p q p q pr q p q pr q q r pqr
q pr pq qr p r pq p r qr q r pqr
p r p q p q p r pq p r q pqr
> + > > + + >
> + > > + + >
> + > + > + >

CHNG MINH
( ) ( )
( ) ( ) ( )
2
2
2 2 2
1, 3
3 0
0
p q
a b c ab bc ca
a b b c c a
>
+ + + + >
+ + >

( ) ( )
( ) ( ) ( )
2
2
2 2 2
2, 3
3 0
0
q pr
ab bc ca a b c abc
ab bc bc ca ca ab
>
+ + + + >
+ + >

MT S PHNG PHP CHNG MINH BT NG THC
BT NG THC SCHUR V PHNG PHP BIN I PQR
Trang 61

( )
3
3
3, 27
27 0
p r
a b c abc
>
+ + >

(ng theo bt ng thc Cauchy vi 3 s)
( ) ( ) ( )
( ) ( ) ( )
2 2
2 2
2 2 2
4, 3 4
( ) 3 4
0
p q pr q
a b c ab bc ca abc a b c ab bc ca
ab a b bc b c ca c a
+ >
+ + + + + + + > + +
+ + >

( )( ) ( ) ( )
( ) ( ) ( )
2 2
2 2
2 2 2
3 3 3
5, 3 4
3 4
0
pq qr p r
a b c ab bc ca abc ab bc ca abc a b c
a b c b c a c a b
+ >
+ + + + + + + > + +
+ + >

( )( )
4 2 2
2 2
6, 3 4
3 0
p q p q
p q p q
+ >
>

(lun ng do
2
3 p q > )
( ) ( )
( )( ) ( )( ) ( )( )
2 2
2 2 2
2 2 2
2 2 2
7, 3 2
3 4 2 3 0
0
p q pr q
p q pr q q pr
ab c a b bc a b c ca b c a
> +
+ + >
+ + + + + >

( ) ( )
( )( ) ( )( ) ( )( )
2 2
2 2 2
2 2 2
3 3 3
8, 2 3
3 4 2 3 0
0
pq p r qr
pq qr p r r p q
abc c a b abc b c a abc a b c
> +
+ + >
+ + + + + >

( ) ( )
3
3
9, 2 9 7
2 4 9 9 0
p r pq
p pq r pq r
+ >
+ + >

(ng theo bt ng thc Schur v bt ng thc Cauchy)
( ) ( )( )
( )( ) ( )( ) ( ) ( )
3 2
3
2 2 2
10, 9 4
9 4 0
0
q r pqr
ab bc ca a b c abc a b c ab bc ca
ab ab bc ab ca bc bc ca bc ab ca ca ab ca bc
+ >
+ + + + + + + >
+ + >

(lun ng theo bt ng thc Schur)
( ) ( )
3 2
3 2
11, 2 9 7
2 4 9 9 0
q r pqr
q pqr r r pq r
+ >
+ + >

( ) ( )
3 3
2 2
12, 6
3 3 0
p r q pqr
pr p q q q pr
+ >
+ >

C.II.3 . V d
V d C.II.1: Cho , , 0 a b c > tho mn 4 ab bc ca abc + + + = . Chng minh rng
( )
2 2 2
3 10 a b c abc + + + >
MT S PHNG PHP CHNG MINH BT NG THC
BT NG THC SCHUR V PHNG PHP BIN I PQR
Trang 62

Chng minh
Ta c
4
4
ab bc ca abc
q r
+ + + =
+ =

( )
2 2 2
2
2
3 10
3 6 10
3 7 6 0(1)
a b c abc
p q r
p q
+ + + >
+ >
>

Ti y ta c nhiu cch lm vi nhiu tng khc nhau nhng chc hn a s s ngh ti
vic lm gim s bin bng cch s dng bt ng Schur hoc s dng mt trong cc bt
ng thc lin h gia p v q. Sau y ti xin gii thiu 2 cch trong tng lm gim s
bin.
Cch 1: S dng bt ng thc Schur
Theo bt ng thc Schur bc 1 ta c
( )
3
3
3
9 4
9 4 4
36
(2)
4 9
p r pq
p q pq
p
q
p
+ >
+ >
+
s
+

T (1),(2) ta cn chng minh
( )( )
3
2
3 2
2
36
3 7. 6 0
4 9
5 27 24 306 0
3 5 42 102 0
3
p
p
p
p p p
p p p
p
+
>
+
+ >
+ + >
>

Mt khc ta gi s 3 p < khi ta c
3
2
1
27
3
3
4
p
r
p
q
q r
s <
s <
+ <

v l
3 p >
Nn bt ng thc trn c chng minh
ng thc xy ra 1 a b c = = =
Cch 2:Ta c
2
2
3
3
p q
p
q
>
s

Nn ta cn chng minh
MT S PHNG PHP CHNG MINH BT NG THC
BT NG THC SCHUR V PHNG PHP BIN I PQR
Trang 63

2
2 18 0
3
p
p
>
>

Ti y lm tng t nh cch 1 ta c pcm
Qua v d trn cc bn hn nhn thy rng cng 1 phng php v cng mt tng
nhng vi cch la chn vic s dng bt ng thc khc nhau s lm cho vic chng
minh sau khi rt bin tr nn d dng hn
V d C.II.2: Cho , , 0 a b c > tho mn 1 abc = Chng minh rng
( )( ) ( )
3 3 3
3
1 1 1
1 1 1 1 a b c
a b c
+ + + + > + + (*)
Chng minh
Ta c
( )
3
3 3 3 3 3 3 3 3 3 3 3 3
3
3 3
3 2
(*) 1
6 8 1
9 6 3 3 0(1)
ab bc ca
a b c a b c a b c a b c
abc
p pq q q
p pq q q
+ + | |
+ + + + + + >
|
\ .
+ + >
+ + >

Do 1 , 3 r p q = >
Nu 3 p q > > ta c
3 2
(1) 3 9 9 6 12 p q q pq q + + + > +
Theo bt ng thc Cauchy ta c
3 2
3 9 9 p q q pq + + >
nn ta cn chng minh
9 9 6 12
3 4
pq pq q
pq q
+ > +
+ >

V
2
3 3 4 p q pq q q > + > + > (do 3 q > )
Nu 3 q p > > theo bt ng thc Schur bc 1 ta c
3
9 4 p pq + > nn ta cn chng minh
2 2
3 3 2
3
q q q
q
> +
>

(lun ng do r=1)
Vy bt ng thc c chng minh
ng thc xy ra 1 a b c = = =
V d C.II.3: Cho , , 0 a b c > tho mn 1 ab bc ca + + = CMR
1 1 1
3(*)
ab bc ca
a b b c c a
+ + +
+ + >
+ + +

Chng minh
Ta c
MT S PHNG PHP CHNG MINH BT NG THC
BT NG THC SCHUR V PHNG PHP BIN I PQR
Trang 64

( )( ) ( ) ( ) ( ) ( )
( )( ) ( ) ( ) ( )
( ) ( )( )( )
( ) ( ) ( )
2
2 2 2
2
2
(*) 1 3
1 1 3
4 3
2 4 3 3
3 3 2 0
1 2 3 0(1)
cyc
cyc
ab c a c b a b b c c a
ab c a b b c c a
a b c abc a b c a b b c c a
p q pr pq r
p p r pr
p p r p
+ + + > + + +
+ + > + + +
+ + + + + + > + + +
+ + >
+ + + >
+ + >


Nu 2 p > ta c (1)lun ng
Nu 2 p < theo bt ng thc Schur bc 1 ta c
3
3
4 9 0
3
9
p pq r
p p
r
+ >

s

nn ta cn chng minh
( )( )
( )( )( )
( )( )
3 2
2 2 3
1 2 0
9
2 5 3 9 0(2)
p p p p
p p
p p p p
+ +
+ >
+ + s

V 2 p < v
2
3 2 3 2
3 27
5 3 9 4 0
2 4
p p p p p p
| |
+ + = + + + >
|
\ .

Nn (2) lun ng
pcm
Du = xy ra 1, 0 a b c = = = cng cc hon v
V d C.II.4: Cho , , 0 a b c > v ( )( )( ) 0 a b b c c a + + + > CMR
( )
( ) ( ) ( )
2 2 2
1 1 1 9
(*)
4
ab bc ca
a b b c c a
(
+ + + + > (
+ + + (


Chng minh
Ta c
( )
( ) ( ) ( )
( ) ( ) ( ) ( ) ( ) ( ) ( ) ( ) ( ) ( )
( ) ( )( ) ( ) ( ) ( )( ) ( )( )( )( )
( ) ( ) ( )
2 2 2
2 2 2 2 2 2 2 2 2
2
2 2 2
1 1 1 9
4
4 9
4 2
9
ab bc ca
a b b c c a
ab bc ca b c c a a b b c a b a c a b b c c a
ab bc ca a b a c b c b a c a c a a b c a b b c c a
a b b c c a
(
+ + + + > (
+ + +
(

(
+ + + + + + + + + + > + + +

+ + + + + + + + + + + + + + + (

> + + +

Theo pqr ta cn chng minh
MT S PHNG PHP CHNG MINH BT NG THC
BT NG THC SCHUR V PHNG PHP BIN I PQR
Trang 65

( ) ( ) ( )
( ) ( ) ( )
2
2
4 2 2 3 2
3 4 2 2
4 4 9
4 17 4 34 9 0
4 9 5 4 6 9 0(1)
q p q p pq r pq r
p q p q q pqr r
pq p pq r q p p q q pr r pq r
(
+ >

+ + >
+ + + + + >

V (1) lun ng nn ta c pcm
Du = xy ra a b c = = hoc , 0 a b c = = cng cc hon v
Nh ta thy 3 v d u khi bi cho cc iu kin rng buc gia cc bin p,q,r
th vic bin i, vic chng minh cng nh nh hng cch lm kh n gin tuy nhin
v d 4 khi khng c iu kin th vic bin i tr nn kh khn hn. gii quyt vn
ny chng ra s dng phng php chun ho c iu kin gia cc bin gip vic
chng minh tr nn d dng hn. Sau y l mt vi v d v phng php bin i pqr
kt hp chun ho.
V d C.II.5: Cho cc s thc khng m a,b,c CMR

( )( ) ( ) ( )
4 4 4 2 2 2 2 2 2 2 2 2
(*) a b c ab bc ca a b c a b b c c a + + + + > + + + +
Chng minh
Do (*) l bt ng thc thun nht bc 6 nn khng mt tnh tng qut ta chun ho
1 ab bc ca + + = Khi ta c
( )( ) ( )( )
( ) ( )( )
( ) ( )
4 4 4 2 2 2 2 2 2 2 2 2
4 2 2 2 2
4 2 2 3
4 2 3
4 2 2
4 2 4 2 2
4 2 4 2 2 4
5 2 4 0
5 4 6 2 3 0(**)
a b c ab bc ca a b c a b b c c a
p p q q pr q p q q pr
p p pr p p r pr
p p p r
p p pr pr p
+ + + + > + + + +
+ + >
+ + > +
+ + >
+ + + >

M theo bt ng thc Schur ta c
4 2 2
4 2
5 4 6 0
5 6 4 0
p p q q pr
p p pr
+ + >
+ + >

V
2
3 3 p q > = nn (**) lun ng ta c pcm
Du = xy ra a b c = = hoc , 0 a b c = = cng cc hon v
V d ***:Cho , , 0 a b c > Chng minh rng
( )( )
3 3 3
4.
5(*)
a b c ab bc ca a b c
b c a c a b a b c
+ + + +
+ + + >
+ + + + +

Chng minh
Do bt ng thc cn chng minh l bt ng thc thun nht bc nn ta chun ho
1 a b c + + = hay 1 p = Khi ta c
1 2 3 4
(*) 5
1 3 3
q r q
q r q r
+
+ >
+

Mt khc ta li c:
1 2 3 4 1 3 3 4
1
1 3 3 1 3 3
q r q q r q
VT
q r q r q q r
+ +
= + > + +
+ +
(do 0 r abc = > )
MT S PHNG PHP CHNG MINH BT NG THC
BT NG THC SCHUR V PHNG PHP BIN I PQR
Trang 66

p dng bt ng thc Cauchy ta c
( )
( )
1 3 3 4
1 3 3 4
2 4
1 3 3 1 3 3
5
q r q
q r q
q q r q q r
VT
+
+
+ > =
+ +
>

pcm
Du = xy ra ( )
3 5 3 5
, , , , 0 ; , , 0
2 2
a b c x x x x
| | | |
+
e
| | `
| |
\ . \ . )
cng cc hon v (vi x l s
dng bt k)
V d C.II.6: Cho , , 0 a b c > Chng minh rng
2 2 2
4 4 4
6(*)
ab bc ca bc ca ab ca ab bc
a bc b ac c ab
+ + + + + +
+ + >
+ + +

Chng minh
Khng mt tnh tng qut chun ho q=3. Khi ta c
( )( )( ) ( )( )( )
( ) ( ) ( )
( )
2 2 2
2 2 2
2 2 2 2 2
2 2 3 3 2 2 3 3 3 2 2 2
2 2 2 3 3 3 3 3
1 1 1
(*) 3 6
1 1 1
2
1 2
2 3
2 2
cyc
cyc cyc cyc
cyc
bc ca ab
a bc b ca c ab
bc ca ab
a bc b ca c ab
bc b ca c ab a bc b ca c ab
b c b c ab a b abc a b c a b c abc a b c
a b c b c abc a b c
+ + + (
+ + >
(
+ + +

+ + +
+ + >
+ + +
+ + + > + + +
+ + + + + + + + + +
> + + + +

(1)
(
(


Theo pqr ta c
( )
( )
2 2 2 3 2 2 3
2 3 3 2
2 2
(1) 2 2 3 3 3 2 3 3
4 2 3 3 3 3
3 36 4 7 0(2)
q pr p q q pr pr q pqr r r r p pq r
r r p pq r q pqr r
p r pr
+ + + + + + +
> + + + +
+ >

Mt khc ta c
2 2 2
3 4 3 36 p q pr q p q pr + > + >
9 3
4 0
3
pq r p r
p
r r
> >
| |
>
|
\ .

Nn ta c
( )
2
8
(2) 3 3 36 4 0
3 3
p pr
p pr r r
| |
+ + + >
|
\ .

Bt ng thc trn lun ng nn ta c pcm
Du = xy ra , 0 a b c = = cng cc hon v
MT S PHNG PHP CHNG MINH BT NG THC
Phng php SS (SchurS.O.S)
Trang 67


D Phng php SS
(SchurS.O.S)
D.I. Gii thiu
Nh bi t mi bt ng thc 3 bin( i xng hoc hon v) u c th bin i c v
dng tiu chun ca S.O.S:
2 2 2
( ) ( ) ( ) 0 S b c S c a S a b
a b c
+ + > . Tuy nhin vic bin i
v dng tiu chun ny thng khng d dng( c bit l vi nhng bi bt ng thc 3 bin
hon v vng quanh). Hn na khi lm bt ng thc bng phng php S.O.S t hng phi
xt 2 trng hp( & a b c c b a > > > > ). y l iu kh bt tin. Phng php SS ra i
cng t nhng bt tin ny gy ra.SS( Schur - SOS ) l phng php a bt ng thc 3
bin thnh dng:
( ) ( )( )
2
M a b N c a c b +
trong & M N l 2 biu thc i xng vi a v b . Nh vy ch cn gi s
( ) max (min) , , c a b c = v chng minh , 0 M N > th bt ng thc c chng minh. Cng
nh S.O.S th phng php SS cng c nhng khai trin rt quan trng:
( ) ( )( )
2
2 2 2
a b c ab bc ca a b c a c b + + = +
( )( ) ( )( )( )
2
3 3 3
3 a b c abc a b c a b a b c c a c b + + = + + + + +
( ) ( ) ( ) ( ) ( )( )( )
2
6 2 ab a b bc b c ca c a abc c a b a b c a c b + + + + + = + +
( ) ( )( )
2
2 2 2
3 a b b c c a abc c a b b c a c b + + = +
( ) ( ) ( )( ) ( )( )
2 2
4 2
cyc cyc
a abc a a b c a b ab a c b c c a c b
(
( = + + + + + +




( ) ( ) ( )( ) ( )( )( )
2
3
2 2
cyc cyc
a b c abc a a c b c a b ab ac bc c a c b + = + + + + +


( ) ( ) ( )( )( )
2
3 2
sym cyc
a b abc a ca cb a b a ac c a c b = + + +


( ) ( )( )
2 1 1
3
a b c
a b c a c b
b c a ab ac
+ + = +
( ) ( )( )
2 2
6
cyc
b c a b
a b c a c b
a ab abc
+ +
= +


Phn
D
MT S PHNG PHP CHNG MINH BT NG THC
Phng php SS (SchurS.O.S)
Trang 68

( )( )
( )
( )( )( )
( )( )
2 3 1 2
2 2
cyc
a a b c
a b c a c b
b c a c b c a b b c a c
+ +
= +
+ + + + + +


( )
( )( )
( )( ) ( ) ( )
( )( )( )
2 2
2
1 1
3
sym
c a c b k k k a k b kc
k a b
a kb
a kc ak c bk c a kb b ka c kb
(
+ + +

+

= +
+ + + + + +


( )( )
( )
( )( )
( )( )( )
( )( )
2 2 2
2
2 2 2
2
2
a b c a b c
b c a c a b
a b c a b c
a b c
a b c a c b
a c b c a b b c c a
+ + =
+ + +
+ + + +
+ +
+
+ + + + +

Nhn chung, t tng ca SS c phn kh ging vi S.O.S, cn cc php bin i ca SS
thng kh hn S.O.S, tuy nhin cc hng lm sau bin i a dng v n gin hn S.O.S.
Chnh v vy chng ta cn s dng linh hot v hp l hai phng php ny.
D.II. Phng php SS vi cc bt ng thc t cc
tr ti tm
V d D.II.1: Cho , , 0 a b c > Chng minh rng
( )
2 2 2
4
2
a b c
b c a c a b
a b c ab bc ca
+ +
+ + +
+ + > +
+ +

Chng minh
Cch 1: Phng php SS
Bt ng thc cn chng minh tng ng vi:
( )
( ) ( )( )
( ) ( )( )
( ) ( )( )
( )
( )
( )
( )
2 2 2
2
2
2
2
6 4 1
2
4 4
2 4 4
0
2
a b c
b c a c a b
a b c ab bc ca
a b c a c b
a b
a b c a c b
ab abc ab bc ca ab bc ca
a b
a b c a c b
ab ab bc ca abc ab bc ca
bc ca ab a b ab bc c
a b
ab ab bc ca
(
+ +
+ + + | |
( + + >
|
+ +
\ . (


+
+ > +
+ + + +
+ | | | |
+ >
| |
+ + + +
\ . \ .
( + + + +
+
(
+ +
(

( )
( )
( )( )
( )
( )
( )
( ) ( )
( )
( )( )
( )
( )
( )
( ) ( )
( )
( )( )
2 2
2
2
2
4
0
2
2
0
2
0 (1)
a abc
c a c b
abc ab bc ca
a b ab c ab a b
bc ca ab
a b c a c b
ab ab bc ca abc ab bc ca
bc ca ab a b c ab a b
a b c a c b
ab ab bc ca abc ab bc c
lllllll
a
(
>
(
+ +
(

(
+ + + ( +
( + >
(
+ + + +
( (


(
( + + +
+ > (
(
+ + + +
( (


Khng mt
tnh tng qut gi s ( ) max , , c a b c = .D dng chng minh c bt ng thc (1) ng.
Cch 2: Phng php S.O.S
MT S PHNG PHP CHNG MINH BT NG THC
Phng php SS (SchurS.O.S)
Trang 69

( )
2 2 2
2 2 2
4
2
2 4 1
cyc
a b c
b c a c a b
a b c ab bc ca
a b a b c
b a ab bc ca
+ +
+ + +
+ + > +
+ +
| | + + | |
+ >
| |
+ +

\ .
\ .



( ) ( )
( )
( ) ( )( )
( )( )
2 2
2
2
2
2 2
2 2 2
2
1 2
0
2 0
0
( ) ( ) ( ) 0
cyc cyc
cyc
cyc
cyc
a b a b
ab ab bc ca
a b
ab ab bc ca
c ab bc ca abc a b
bc ac abc a b
S b c S c a S a b
a b c

>
+ +
| |
>
|
+ +
\ .
+ + >
+ >
+ + >


trong :
2 2
2 2
2 2
S ba ca abc
a
S cb ab abc
b
S bc ac abc
c

= +

= +

= +


Gi s a b c > > ( trng hp a c b > > ta lm tng t).Theo iu kin 2: a b c > > v
, , 0 S S S S S
b b a c b
+ + >
th bt ng thc c chng minh).
V d D.II.2: Cho , , 0 a b c > Chng minh rng

( ) ( ) ( )
2 2 2
2 2 2
a bc b ac c ab a b c
b c a c a b
b c c a a b
+ + +
+ + > + +
+ + +
+ + +

Chng minh
Bt ng thc cho tng ng vi:
( ) ( ) ( ) ( ) ( ) ( )
( )
( )( )
( )
( )( )
( )
( )
( ) ( )
2 2 2
2 2 2 2 2 2
2
2
2
2 2 2
0
0
0
cyc
cyc
a bc b ac c ab ab ac ba bc ca cb
b c c a a b b c c a a b
a bc ab ac
b c
a b a c
b c
c a c b
a c b c
a b
a b b c c a
+ + + + + +
+ + > + +
+ + + + + +
+
>
+

>
+
(


+ > (
+ + + (


MT S PHNG PHP CHNG MINH BT NG THC
Phng php SS (SchurS.O.S)
Trang 70

( )( )
( )
( )
( )( ) ( )( )
( ) ( )
( )( )
( )
( )
( ) ( )
( ) ( )
2 2 2 2
2 2 2
3 2 2 3 3 2 2 3
2 2 2
0
0
a c a c b c b c
c a c b
a b
a b b c c a
a a c ac c b b c bc c
c a c b
a b
a b b c c a
(
+ +

( + >
( + + +

(
+ +

( + >
( + + +


( )( )
( )
( )
( ) ( ) ( )
( ) ( )
( )( )
( )
( )
( ) ( )
3 3 2 2 2
2 2 2
2 2 2
2
2 2 2
0
0 (1)
a b c a b c a b
c a c b
a b
a b b c c a
c a c b
a ab b ca cb c
a b
a b b
a
c
a
c a
a a
(
+

( + >
( + + +


+ + + +
+ >
+ + +

Khng mt tnh tng qut gi s ( ) min , , c a b c = . D thy (1) ng.
Vy bt ng thc c chng minh. Du bng xy ra khi a b c = =
V d D.II.3: Cho , , 0 a b c > Chng minh rng
( )
( )
2
2 3 3
2 11
3
sym
a b
abc
a b c
b c
3
+
+ >
+ +
+


Chng minh
Bt ng thc cho tng ng vi:
( )
( )
2
2 3 3 3
2 2
3 0
3
sym
a b
abc
a b c
b c
+
+ >
+ +
+


Mt khc:
( )
( )
( )
( ) ( )
( )
( )( )
( ) ( )
( )( )
2 2
2
2 2 2 2 2
2 2 2
3
sym
a b a b c a b c a b c
a b c a c b
b c a b b c a b b c
+ + + + + + +
= +
+ + + + +

(1)
( )
( )
( ) ( )( )
2
3 3 3 3 3 3
2
2 2
3 3
a b c
abc
a b c a c b
a b c a b c
+ +
(
= +

+ + + +
(2)
T (1)&(2) ta c:
( )
( )
( )
( ) ( )
( )
( )
( )
( )( )
( ) ( )
( )
( )
( )( )
( ) ( ) ( )( ) ( ) ( )
( )( ) ( )( ) ( )
2
2 3 3 3
2
2
2 2 3 3 3
2 2
3 3 3
2 2 2 2
3 3 3
2 2
2 2
3 0
3
2 2
3
2 2 2
0
3
3 2 2
3 2 2 2
sym
a b
abc
a b c
b c
a b c a b c
a b
a b c a b b c
a b c a b c a b c
c a c b
a b c a b a c
a b c a b c a b c a b b c a b
a b c a b c a b c a b a c
+
+ >
+ +
+
(
+ + + +
(
+ + ( + +

(
+ + + + + +
( + >
+ + ( + +

(
+ + + + + + + + +

+ + + + + + +

( )( ) 0 c a c b
(
>


Khng mt tnh tng qut gi s ( ) max , , c a b c =
MT S PHNG PHP CHNG MINH BT NG THC
Phng php SS (SchurS.O.S)
Trang 71

Ta cn chng minh:
( ) ( ) ( )( ) ( )
( )( )( ) ( )( ) ( )
2 2 2
3 3 3
2 2
3 3 3
3 2 2 0
3 2 2 2 0
a b c a b c a b c a b b c
a b c a b c a b c a b c a b a c

+ + + + + + + + >

+ + + + + + + + + + >


Tht vy:
( ) ( )
( ) ( )( )( )
2
3 3 3
2 2 2
3 2
4
a b c a b c
a c b c a b c a b c
+ + + +
> + + + + + +
>

( )( )( ) ( )
( )( )( ) ( )
( )( )( )( )
( )( ) ( )
( )( ) ( )( ) ( )
( ) ( ) ( )( ) ( )
( )( ) ( )( ) ( )
2 2 2
2 2 2 2
2
2 2
2 2
2 2 2
3 3 3
2 2
2 2
2
2
2
3 2 2 2 0
3 2 2 0
3 2 2 2 0
a b c a b b c a b c
a b c a b b c a b c b
a b c a b b c ab ac bc b
a b c a b b c
a b c a b c a b c a b a c
a b c a b c a b c a b b c
a b c a b c a b c a b a c
> + + + + + +
(
> + + + + + + +

> + + + + + + +
= + + + +
+ + + + + + + + >

+ + + + + + + + >

+ + + + + + + + >


D.III. Phng php SS vi cc bt ng thc cc
tr khng t ti tm
Phng php SOS v SS t ra kh hiu qu vi cc bt ng thc 3 bin khng cha
cn, nhng mt yu t kin quyt a v dng chun ca phng php l bt ng thc
phi c du bng t ti tm. Tuy nhin vi nhng bt ng thc du bng khng xy ra ti
tm, chng ta cng vn c th p dng phng php SS chng minh. Cc bi ton sau y
s minh ha cho k thut ny.
V d D.III.1: Cho
, , 0
3
a b c
a b c
>

+ + =

. Chng minh: ( ) ( ) ( )
2 2 2
27
4
a b c b c a c a b + + + + + s (1)
Chng minh
( ) ( ) ( )
( )
( ) ( ) ( ) ( ) ( )
3
2 2 2
3
2 2 2
(1)
4
4 (2)
a b c
a b c b c a c a b
a b c b c a c a b a b c
+ +
+ + + + + s
+ + + + + s + +

ng thc khng xy ra ti tm m ti
3
, 0
2
a b c = = = v cc hon v nn nu ta gi nguyn
bt ng thc nh trn th kh c th bin i v dng chun ca phng php SOS v SS.
Chng ta s xut hin tng cng thm mt biu thc khng m vo v tri ca bt ng
thc (2) c thm iu kin ng thc xy ra ti tm 1 a b c = = = nhng vn phi bo ton
iu kin ng thc cng xy ra ti
3
, 0
2
a b c = = = . i lng cng thm l kabc. Cho a
= b = c = 1 ta s c k = 3. Gi s c = min (a , b, c) ta s chng minh BT mnh hn
MT S PHNG PHP CHNG MINH BT NG THC
Phng php SS (SchurS.O.S)
Trang 72

( ) ( ) ( ) ( )
3
2 2 2
4 3 a b c a b c b c a c a b abc ( + + > + + + + + +

(3)
( ) ( ) ( ) ( )
( )( ) ( )( )
3 3 3 2 2 2 2 2 2 2 2 2
2
3 6 4 3
0
a b c a b ab a c ac b c bc abc a b c b c a c a b abc
a b c a b c a c b c
( + + + + + + + + + > + + + + + +

+ + >
iu ny ng do c = min (a,b,c)
Bc tinh t nht trong li gii trn chnh l tng chuyn t BT (2) v BT (3).
Cng vic tng chng lm cho BT cht hn li gip to BT cc tr t ti tm c
th chng minh bng phng php SS hay SOS. i vi nhng bt ng thc c du
bng xy ra ti bin, i lng cng thm vo thng c dang k.abc.f (a, b, c) m
bo c du bng BT ban u vn xy ra ti bin v du bng BT mi c xy ra ti
tm. T chng ta c th thy, vic tm f (a, b, c) thch hp chnh l nt tinh t, cng
nh kh nht ca vic bin i ny.
V d D.III.2: Cho
, , 0
2
a b c
a b c
>

+ + =

. Chng minh rng


2 2 2 2 2 2
1 a b b c c a + + s (1)
Chng minh
( ) ( ) ( )
4
2 2 2 2 2 2
(1) 16 8 a b c a b b c c a abc a b c + + > + + + + +
Ta s chng minh
( ) ( ) ( )
( ) ( ) ( ) ( )
( ) ( )
( )
4
2 2 2 2 2 2
4 4 4 2 2 2 2 2 2 3 3 3
2 2 2 2 2 2
4 2 2 3
16 11
( ) 6 4 ( ) 12
16 11
10 4 2
cyc cyc cyc cyc cyc cyc
a b c a b b c c a abc a b c
a b c a b b c c a a b c b a c c a b abc a b c
a b b c c a abc a b c
a abc a a b abc a a b c abc a
+ + > + + + + +
( + + + + + + + + + + + + + +

> + + + + +
( | | | |
+ +
| | (
\ . \ .

( ) ( ) ( ) ( )
( ) ( ) ( )( )
( ) ( )( )
2 2
2 2
2
0
10 4
10 8 4 4 0
. . 0
a b a b c c a c b c
c a c b ab a c b c ab ab ac bc
M a b N c a c b
>
(
+ + + + + +

+ + + + + + > (

+ >
Vi
2 2 2
2
6 4 4 5
5 5
M a b ab ac bc c
N ac bc c
= + + + +
= + +

D thy vi ( ) min , , c a b c = th bt ng thc c chng minh
V d D.III.3: Chng minh rng:
( ) ( ) ( )
2 2 2
2
a b c b c a c a b
a bc b ca c ab
+ + +
+ + >
+ + +
, , 0 a b c >
Chng minh
Ta s chng minh
( ) ( ) ( )
( )( )( )
2 2 2
2 2 2 2 2 2
8
2
a b c b c a c a b a b c
a bc b ca c ab a bc b ca c ab
+ + +
+ + > +
+ + + + + +

MT S PHNG PHP CHNG MINH BT NG THC
Phng php SS (SchurS.O.S)
Trang 73

( )( )( ) ( )( )( )
( ) ( )
2 2 2 2 2 2 2 2
4 2 2 2 3 3 3 2 2 2
2 8
3 2 12
sym
sym sym sym sym
a b c b ac c ab a bc b ac c ab a b c
a b c abc a b c b c abc a a b c
(
+ + + > + + + +

+ + + + > + +



( ) ( ) ( ) ( )( )( )
2 2
2 2
4 2 2 0 ac bc ab c ab c ac bc a b abc a b a c b c
(
+ + + + + >


Gi s ( ) min , , c a b c = bt ng thc trn ng, ta c iu phi chng minh
V d D.III.4: Chng minh
2 2 2 2 2 2
4 1 1 1
, , 0
5
a b c
a b c
b c c a a b b c c a a b
| |
+ + > + + >
|
+ + + + + +
\ .

Chng minh
Ta s chng minh bt ng thc mnh hn:
( )( )( )( )
2 2 2
2 2 2 2 2 2 2 2 2 2 2 2
4 1 1 1 36
5 5
a b c a b c
b c c a a b b c c a a b a b b c c a a b c
| |
+ + > + + +
|
+ + + + + + + + + + + \ .

( )
( )( )( )
2 2 2 2
2 2 2 2 2 2 2 2 2 2
12 36
5 5
sym sym sym
a b c a a a b c
b c b c b c a b b c c a
+
+ > + +
+ + + + + +


Ta c th chng minh c cc bt ng thc sau:
2
2 2
4 4
5 5
sym sym
a a
b c b c
>
+ +

(1)
( )( )( )
2 2 2 2
2 2 2 2 2 2 2 2
1 4 1
.2
5 5
sym
a a b c
b c a b b c c a
| |
| + >
|
+ + + +
\ .

(2)
( )
( )( )( )
2 2 2
2 2 2 2 2 2 2 2
8
2
sym
a b c a b c
b c a b b c c a
+
> +
+ + + +

(3)
Cng 2 v ca (1), (2) v (3) ta c iu phi chng minh.
Phng php S.S t ra kh hiu qu ngay c trong cc bt ng thc du = khng
xy ra ti tm. Tuy nhin, mt nhc im ca phng php ny l vic bin i kh
phc tp do khng mang tnh i xng nh phng php S.O.S Vic bin i thnh dng
chun ca phng php l mt k nng ht sc quan trng. Bn cnh , vic chng
minh M v N dng trong cc bi ton cng khng h n gin, thng yu cu nhng
chng minh kh di, c th dng o hm Tuy nhin, xt v tng th, phng php S.S
c ng li r rng v chng minh c rt nhiu bt ng thc kh. V vy, y l mt
phng php mnh bn cnh cc phng php khc, lm phong ph thm cc cch
chng minh bt ng thc hin i.

Trang 74

LI KT
y l sau 1 thng thc hin, chuyn ca chng ti hon
thnh. Mi ngi u rt tm c vi kt qu lm vic ny ca
mnh. Mt ln na, chng em xin gi li bit n su sc n thy gio on
Thi Sn v nhng ngi gip chng em hon thnh chuyn ny.
Nh chng ti ni, bt ng thc l mt trong nhng mng kin thc
kh v rt a dng ca ton hc. Cc phng php trnh by trong chuyn
ca chng ti khng phi l nhng phng php mu, ch l mt trong
nhng hng i trong v vn cch tip cn ca cc bi ton bt ng thc.
Chng ti hy vng sau khi c xong chuyn ny, cc bn s vn dng mt
cch sng to nhng phng php c trnh by vo gii ton t kim
chng sc mnh ca tng phng php v c bit l s tm ti khm ph ra
nhng phng php mi c o cho ring mnh.
Trn Tin t v cc cng s
Lu Trung Kin Nguyn Hng c Th Hng Vn
V



Mt s phng php chng minh
bt ng thc
Phng php S.O.S
Phng php dn bin
Phng php P.Q.R
Phng php S.S
----------------------------------------
Trn Tin t
Lu Trung Kin
Nguyn Hng c
Th Hng Vn
Lp 10 Ton trng THPT chuyn Trn Ph Hi Phng
Thng 5/2009

You might also like